Tanveer Notes Revised 1st ED (Uploaded by Hanan)

April 28, 2017 | Author: jawad | Category: N/A
Share Embed Donate


Short Description

Download Tanveer Notes Revised 1st ED (Uploaded by Hanan)...

Description

Please don’t make any errata of these notes. If you find any mistakes kindly tell us with authentic reference and we will correct them for you. This way everyone will benefit from it. Keep us in your prayers.

Written by Dr Tan Veer Ahmad Compiled and Edited by Dr Muhammad Hanan yousaf

Session 1

1

in case of mistakes, if any, feel free to ask at my e-mail ….. 1.Patient with bone pains having normal Calcium,Inc Alkaline phosphatase most likely suffering from = Paget’s Disease ●Osteoporosis = Labs are normal ●Pagets disease = only ALP is raised ●Osteopetrosis = Only Ca is dec ●Osteomalacia = Vit D dec → dec Ca & phosphates → inc PTH & ALP 2.Erythropoises occurs in middle trimester mainly from = liver (Regarding Fetal erythropoiesis ●3 to 6 weeks = Yolk sac ●6 weeks till birth = Liver ●10 to 28 weeks= Spleen ●18 weeks to adult = Bone Marrow) 3.Vertebrae is derived from = Sclerotome, 4.Memory center is located in = Temporal lobe ( note ; for Dejavu = temporal lobe) 5.The most common type of salivary gland tumor is = Parotids 6.Distruction of Ant.pituatary gland causes dec.functioning of = Glom.fasiculata ( Remember the Mnemonic GFR ie ●Glomerulosa → aldosterone and this one is under the control of Angiotensin not Fasiculata → Cortisol and this one is under the control of pituitary ●Reticularis → Sex steroids ) 7- A normal looking Girl came to you with primary amenorrhea, on examination she was having short blind vagina wid normal vulva, scanty pubic n axillary hairs, normal breasts, absent uterus, her karyotype is = 46XY ( blind vagina and absent uterus always mean XY until proved otherwise ) 8-Nerve supply to extensors of arm is supplied by = Post cord (Radial nerve is from posterior cord ) 9. Nerve injured in Ant dislocation of Shoulder joint is = Axillary 10-Damage to scaphoid bone causes injury to = Radial Artery 11-Common site of fracture at rib is = Angle 12-Following is not a tumor marker = Acid Phospatase

Session 1

2

13-Most common histology found in lung tumors is = Adenocarcinoma 14-Patients comes with Rt. deviation of tongue, Dec.sense of touch and vibartions,the artery commonly involved in brain is = Ant.Spinal artery ( whenever u see deviation of tongue in such a scenario, look for ant spinal artery in options ) 15-Soldier comes with heavy bleeding,the ideal fluid replacement would be = Whole blood for 3 days ( same for same, ie for heavy blood loss, prefer blood ) 16-Maxillary sinus opens into = Middle meatus 17-Patient having increased levels of IgE,most probably suffering from = Parasitic infection 18-Autospy done on the patient having CRF,most prabable finding would be = Hypertrophied PTH gland 19-Excitation of hypothalamus would cause = Shivering (note for hypothalamus remember that ANTERIOR is for cooling and POSTERIOR is for heating ANTERIOR is parasympathetic center while POSTERIOR is sympathetic) 20-Foetal period starts after which week =8th (3-8 weeks = Embryonic period, this is most sensitive to teratogens, while fetal period starts after week 8) 21-Patient having dec levels of factorII,VII,IX,X,most likely have = Dec Vit.K ( memorize vit K DEPENDENT FACTORS by 1972 ie factor 2,7 .9 ,and 10 consider 1 as = Factor 10 ) 22-Young pt having Hb:6.0,TLC:3.5/ul,Plts:1lac is suffering from = Aplastic anemia 23-Nucleus ambigus supplies = Larynx (Nucleus ambigus is for IX,X,XI ie cranial nerves 9, 10, 11 also remember that the parasympathic nucleus for vagus nerve is DORSAL NECLEUS, ASKED REPEATEDLY IN PAST PAPERS) 24-Olfactory area is present in = temporal gyrus ( this the only sensation that doesn’t pass through thalamus ) 25-Patient with injury to left 8th cervical segment of spinal cord will not show following sign = Dec sense of pain and temperature below the lesion on same side (pain and temp loss are contralateral ie opposite to the lesion , while position and vibrations senses are ipsilateral) 26-Feature of Rickets and Osteomalacia is = Decreased bone density 27-Most common site of malignancy in pts suffering from nuclear outbreak = Hematopoietic (don’t confuse it with head and neck radiations that causes

Session 1

3

papillary thyroid cancer) 28-Virus acts on cells by = Involving protein synthesis 29-Most sensitive cells to hypoxia are = Neurons 30-Myelin in CNS is formed by = Oligodendrocytes 31-Feature not associated with irreversible cell injury is = Myelin figures (For cpsp always remember Myelin figure = Reversible injury) 32-Ureter is not constricted at = Psoas muscle 33-Common relation of ureter is = post.to gonadal vessels and Ant.to iliac Artery 34-Patient feels pain after she underwent surgery for fibroids, structure injured would be = Ureters 35-Least common site for ectopic pregnancy would be at = Ovaries 36-Gall bladder is supplied by = Only Cystic Artery 37-Venous drainage of urinary bladder is into = Internal Iliac Vein (those who appear in Gyne exam, should cram the whole anatomy of urinary bladder) 38. Patient with fluid loss now show metabolic acidosis e hypokalemia. most fluid loss will be through = Colon ( colon is reservoir of HCO3 & POTASSIUM, and in diarrhea both are lost resulting in acidosis and hypokalemia 39-Patient having hyperplastic bone marrow, anaemia, Increase Platelets is probably suffering from = Leaukemia 40-Stimulus for Erythropoietin secretion is = Hypoxia 41-Erytropoiten is always raised in = Sec.polycythemia (Polycythemia vera is only type of polycythemia in which there is Dec. EPO) 43-Patient after gastrectomy is on VitB12,the cells lost in the gastrectomy = parietal 44-Fat tags attached to the walls of large intestine are known as = Appendeces epiplocea 45-Bile salts are reabsorbed from the = Ileum 46-Following is an autosomal dominant disease is = Hereditary spherocytosis 47-Type of defect in Hereditary spherocytosis = Structural defect 48-Patient with jaundice, anaemia,high reticulocyte count is having = Hemolytic anaemia 49-Ejaculatory duct opens into = Prostatic urethra 50-Terminal part of CBD in relation to pancreas is = Embedded into pancreas 51-Following would cause massive infarction and destruction leading to patient death = Thromboembolism 52- A 22yrs female pt.with 13th week of pregnancy after having crush fractures

Session 1

in RSA dies after 3 days, most likely cause of death is = Fat embolism (Note that if fracture and 3 days duration are not mentioned then better option will be amniotic fluid embolism ie it causes immediate deaths) 53-Patient with old history of adenoCA of colon is operated for polypectomy,on histologic evaluation pathologist labelled it as benign growth with no chances into malignant transformation,it would be = Tubular adenoma 54-Tyrosine derivative doesn’t include = Prolactin 55-Precursor of steriod hormone is = Cholesterol 58-Wasting of thenar eminence,loss of sensation over thumb and index finger,nerve involved is = Median ( Regarding wasting of the thenar eminence always click median nerve) 59-The base of urinary bladder is made by = Post surface 60-Shock without having vasodilation is likely associated with = Burns 61-Following doesn’t form the layer of inguinal canal = rectus abdominus 62-Femoral sheath is formed by = Fasica transersalis and Fasica iliaca 63- The muscle involved in unlocking of knee joint is = Poplitus, 64-External spermatic fascia is formed by: = External oblique 65-In Turner syndrome, the genotype would be = XO 66-Organ having least chances of infarction = Liver 67-Type of necrosis in brain infarction = Liquefactive 68-Pt.with cervical lymphadenopaty,+ve AFB sputum,sign to look for TB on microscopy is = caseous necrosis 69-Thyroid gland moves with swallowing because it is enclosed in = Pretracheal fasica 70-Neurovascular bundle in chest wall lies between = Internal and Innermost layers 71-Fibrous pericardium and mediastinal pleura is supplied by = Phrenic Nerve ( Remember the Mnemonic i-e ●Fibrous pericardium = Phrenic (frenic) nerve ● Visceral pericardium = Vagus nerve) 72-Man is suffering from testicular ca,the lympahtic drainage of testis is into = Para aortic Lymph nodes 73-In exercise,the venous blood returns to the heart by = Muscle pump in calves 74-Circulation in heart is maintained by = Local metabolites 75-Fisherman is presented with gingival bleeding and ecchymosis,he is suffering from = Vit C deficiency ( NOTE ; In vit B12 def due to Diphylobotram Latum among the fishermen,

4

Session 1

5

they won’t mention points like gingival bleed and ecchymosis) 76-Factor affecting collagen synthesis during healing is = Vit C 77-Mesothelioma is associated with = Asbestos 78-Child having meckels diverticulum is having bleeding per rectum,it is due to = Ectopic gastric tissue 79-During lactation,Amenorrhea is due to = Inhibition of LH and FSH 80-Presence of pancratic tissue in gastric mucosa is termed as = Choristoma ( The other site for choristoma is Brain tissue in Nasal cavity ) 81-Main difference between primary and secondary intention wound healing is = Wound contraction 82-Child with sore throat after 3 weeks developed oedema,haemturia,cause would be = post streptococcal GN 83-Microscopic feature of polyarterites nodusa is = Fibrinoid necrosis (for any vasculitis remember fibrnoid nerosis, for brains liqufactive and for solid organs remember coagulative necrosis , TB also has a type of coagluative necrosis) 84-Angiotension II exerts its effects by activating = Aldosterone 85-Tumor compressing optic chiasma would cause = Bitemporal haemianopia 86-Growth hormone is maximally rasies in = Exercise ( ●For max growth hormone sec NREM sleep > Exercise > REM ● if only sleep is mentioned then prefer Exercise and if NREM sleep is mentioned then click it) 87.Lymph drainage of perianal skin is from one of the following = medial group of horizontal superficail nodes 88.Lesser omentum = .connects with deudenum 89.Which of the following is not a feature of shock = increased urine output 90.Which is most imp indicator of muscle protein loss = urinary nitrogen 91. what is true about DNA = .Double stranded in which two strands are antiparallel to each other 92. Least chances of renal stones is associated with = Hyperlipidemia 93.A pt came to you with primary amenorrhea, webbed neck, short stature diagnosed as Turner syndrome, what is her karyotype = A.45XO 94.supply of sphincter urethra comes from = .Pudendal nerves 95.Thymus develops from = endoderm of 3rd brachial pouch 96.A pt presented with an opening in the ant wall of sternocledomastoid muscle with pussy discharge coming out of it.this problem occurred due to abnormality in one of the following = 2nd pharyngeal cleft

Session 1

6

97.Diaphragmatic hernia occurs due to = absence of pleuro-peritoneal membrane 98. Adult derivative of notochord is: = Nucleus pulposis 99.Superior parathyroid gland develops from = 4th brachial pouch 100.In cervical carcinoma,what is the lymph drainage = both internal & external iliac 101. Cervical rib attached to transverse process will compress = T1 (though it can Compresses both C8 and T1 ) 102. Aortic aneurysm which structure not compressed = Phrenic nerve 103. Resting membrane potential achieved by =Diffusion of potassium outside the cell (Achieved by K efflux, maintained by N-K ATPase Pump while depolarization is maintained by Na influx) 104. Liver weight = 1.5 kg (1.44-1.66 kg is Range) 105. What is median if values are 20 20 25 30 35 40 25 30 = 27.5 (Arrange in Ascending order 20 20 25 25 30 30 35 40 than take middle two values 25+30/2=27.5) 106.. Dead organisms in vaccine = Pertussis (Pertusis, Cholera, Influenza ,Inj Polio & Rabies all Killed vaccines while Tetanus is Toxoid and MMR is Live attenuated) 107.Esophagectomy done , stomach is mobilized to upper portion of thorax, To anastomosis with pharynx Which artery is responsible for stomach supply = Right gastroepiploic 108. Patient is pregnant and suddenly faints. She'll be positioned in left lateral and not right lateral to avoid compression to which abdominal structure = IVC 109. Post op wound having greenish pus discharge organism involved = Pseudomonas 110. Prevalence is = All cases (Prevalence: All current cases (old & new) while incidence is just new cases) 111. Common carcinogen of CA Cervix = HPV 112.Apex of the heart = 8 cm from median plane 113.After normal delivery in primigravida the alive n healthy new born is given feed = After half hour ( Nelsons states 1 hour, but prefer half hour if in options ) 114. Diagnostic tumor marker for ovarian CA = CA 125 115. Which of the following prolong use of drug will cause persistent increase in Gastrin secretion = PPI

Session 1

7

116. A pt presented to his dentist due to inability to put his dentures, dentist referred back to physician saying there is no dental problem, where is the lesion = Substantia nigra 117. Chronic Myeloid Leukemia = Chromosome 9 & 22 118. Steroids drug action on cells = Act on Genes (all steroids & thyroxin act on genes) 119. ADH is inhibited by = Alcohol 120. A tumor with all germ layers = Teratoma 121. Following has high clearance value = PAH 122. Scanty bar bodies = Turner (REMEBER IF ASKED ● Bar body diagnostic of then = klienfilter ● scanty bar body or no bar body then = turner ● bar body TEST is diagnostic for = turner) 123. Which parasite is not transmitted thru orofecal route = Trichenella Spiralis (Humans get it when they eat infected meat/pork ) 124. Organelle having microtubules in its structure = Centriole 125. S1 nerve root lesion = Loss of Ankle jerk 126. Mucus not secreted by = Terminal bronchioles 127. Which of the following increases lower esophageal tone n speeds up gastric emptying time = Metoclopramide 128. Diphtheria vaccine reaction = type 3 ( remember any serum will cause serum sickness ie type 3 hypersensitivity ) 129. Femoral nerve derived from which section of spinal cord = L2 L3 L4 130. Deep coma is caused by lesion of which nucleus = Nucleus cerelous (REMEMBER IF ASKED ● Deep coma then nucleus cerelous ● Prolonged coma then Periq aquiductal gray matter ( p for prolonged & p for periquiductal & ●permanent coma = thalamus ) 132. A man of 70 has recurrent UTI cause = Outflow Obstruction (old age →chances of BPH → lead to outflow obstruction→ leading to recurrent UTI) 133. Normal FEV/FVC1 Ratio = 0.8 134. Compression effects on esophagus by which organ enlargement = Left Atrium 135. Plasma Protein Binds With All Except = Oxygen

Session 1

8

136. Patient presented in emergency, deeply comatosed,not responding to nalaxone: = Phenobarbital 137. Cause of Mental retardation = Fragile X Syndrome ( if down syndrome is mentioned then click it, because Fragile X syndrome is the 2nd most common cause) 138. In IV drugs abusers = Bacterial Endocarditis ( STAPH ARUEUS) 139. Highest bioavailability of anti-emetics = Metacloprmide 140. X.ray of elbow of 12 yr girl, which Secondary ossification center will appear last on x-ray = Lateral Epicondyle (capitellum > radial head > medial epicondyle > trochlea > olecranon > lateral epicondyle → 1 , 3 , 5 , 7 , 9 , 11 (ODD Numbers so 11 is lateral epicondyle) 141. Baby born on 4th day most common cause of Meningitis = E.Coli (REMEBER : in newborns, GROUP B STREP GROUP ie strep agalactea IS ON THE TOP) 142. A tall man with normal external genitalia married but had no kids he had normal ejaculation period but semen analysis showed azoospermia cause may be= Sertoli cell 143. Pregnant lady anemic, Best lab investigation = CBC (IF ASKED IRON DEF ANEMIA THEN SERUM FERRITIN, TIBC may give false value, cz its affected by pregnancy) 144. Highest percentage of protein in = HDL (NOTE THE POINT; IF ASKED HIGHEST THEN ●PROTEIN = HDL ●TGs = CHYLOMICRON > VLDL ●CHOLESTEROL = LDL) 145. Soft Keratin with no nucleus found in = Stratum corneum 146. A newly married female came with pain in rt illiac fossa her TlC 12000 she has fever n vomiting her lmp was 3 weeks back what is diagnosis = Acute Appendicitis 147.Open wound healing includes = Contraction of myofibroblast 148. Patient aged 65 years, is diagnosed to have severe aplastic anemia. HLA compatible sibling is available. The best option of treatment is: = Anti thymocyte globulin followed by cyclosporine ●ATG & CYCLOSPORIN = 1st line for non-severe aplastic anemia, or severe aplastic > 40 years of age or sever aplastic who have no HLA identical sibling donor

Session 1

9

●HSCT (transplant) = 1st line for the patients who have sever aplastic and have HLA MATCHED sibling donor 149. Most common S/E of use of histamine 1 blocker = Sedation 150. Decrease Insulin will = Ketogenesis in Liver 151. An obese man with h/o atherosclerosis underwent laparotomy due bleeding in abdomen and during surgery his small gut was purple n sup mesenteric vein was ok so what is dx = Wet gangrene 152. First heard sound is produced during = Isovlumic contraction 153. A patient came with c/o bitmep hemianopia so a blockage was seen at = Upper part of optic chiasma ( best ans is optic chiasma, but if both upper and lower part are in options, prefer the upper part of optic chiasma ) 154. 30 yr male newly diagnosed with HTN, on ECG lt ventricular hypertrophy, serum cholesterol 200, FBS 88 mg%, urinary sugar ++ RFTs were normal, K 2.5 , Na 140, what is cause of his HTN = Primary hyperaldosteronism 155. Man With Weakness And Dysphagia, Blood Profile With Target Cells = Iron Deficiency Anemia 156. Which of the following in maternal serum for rubella should b an indication for termination of pregnancy = Ig M 57. Epithelium of Conjunctiva = Stratified Columnar Epithelium ( nonkeratinized ) 158. Antioxidant Vitamin = vit E (REMEMBER IF ASKED ●Potent one then = Glutathion > vit E ●among vitamins then = vit E > vit C) 159. Which of the following does not increase in stress for surgery = Insulin 160. Monitoring of HEP B virus can be done via = SGPT (or ALT) 161. Diseased ovary pain radiating to medial thigh of body due to nearby presence of = Obturator Nerve 162. Biguanides side affects = Diarrhea and flatulence 163. Patient with Gastrectomy presents with Anemia after One and half year, what will you give = Vit B12 164. Data is collected for some clinical trial based on presence or absence of Vomiting. What type of data it is = Nominal (whenever asked about nominal remember = it’s the data type where u have to ans between the two variables like YES OR NO, RIGHT OR WRONG , PRESENCE OR ABSCENCE etc while ordinal gives u order like PRIMARY > SECONDARY > TIRTIARY)

Session 1

10

165. Aphasia (Non-fluent) after stroke or Trauma which area of Brain is involved = Brocas (fluent aphasia is due to wernickes area involvement) 166. What cytoskeletal structure connects Extracellular matrix to Intracellular = intermediate filaments (Also remember that C for Cadherin and C for cell; so it mediates cell to cell attachments and is important in case of metastasis) 168. Cardiac defects in fetus of mother suffering from rubella infection will occur if = 5 th to 10th week 169. Dorsal Column Medial Leminsical carries = Proprioception 170. Contralateral loss of pain and temp at T9 and ipsilateral loss of proprioception below T8, bilateral normal reflexes = Lesion of spinothalamic tract at T8 ( In these scenarios always decide the level of lesion by the level at which there is loss of peropioception like in this case T8 is given so lesion should be at T8 ) 171. Downs Syndrome = Increase incidence with increasing mother age 172. Pulmonary vasoconstriction occurs due to = Reduced systemic PO2 ( this concept is repeatedly asked in many ways but the basic concept remains the same ie HYPOXIA CAUSES VASOCONSTRICTION IN PULMONARY VESSELS) 173. Most imp buffer in blood = HCO3 174. Gastrin secretion is inhibited by = Somatostatin 175. Which vitamin act as cofactor in oxidation of fatty acids = Biotin 176. Oxytocin and vasopressin originated from = Hypothalamus 177. If FACTOR IX not available = FFP (cyropreciptate is for factor 8, so to memorize pronounce it as CRYOPRICEPIT-8) 178. All of the following will cause lesion at genital tract except = Gonorrhoea 179. Vagus decreases HR by which mechanism = Na (permeability) (The mechanism is , parasymp system decreases Na CHANNELS INWARD CURRENT THAT IS RESPONSIBLE FOR PHASE 4 DEPOLARIZATION IN SA NODE ) 180. Maximum absorption of water = Jejunum 181. Patient with beta blocker ECG show increase PR = Drug induced block 182. Drug causing peripheral neuro and pancreatits = Diadenosine 183. Left Kidney is not related to = Diaphragm 184. What age the head of fetus become exactly half the length at CRL = 3rd month 185. Prostatic carcinoma spread to skull bones through = Vertebral plexus

Session 1

11

186. First heart sound = Isovlumic contraction 187. Pregnant women 1st trimester develops Atrial fibrillation what would u give = IV Heparin 188. The tumor which is the ovarian counter part of seminoma of the testis is = Dysgerminoma 189. Patient came in with sore throat, fever and lymphadenopathy. His monspot test was positive. His blood shows atypical leukocytosis. What is the type of cell that is affected = B-cell (REGARDING INFECTIOUS MONONUCLEOSIS IF ASKED ●Type of cell affected then = B CELLS ●Atypical cells that proliferate in response then = T CELLS) 190. Pt was brought into the Emergency Department and his family wanted to accompany him. What is the most appropriate response = Only pt should be allowed in the ER 191. A pregnant full term patient came into the ER with pre-eclampsia. Her Blood pressure was 190/110. She was admitted for emergency ceserean section. What is the best medicine to bring down the blood pressure = Hydralazine 192. Patient was admitted in the ICU. HE had a central venous catheter for 10 days. HE started having fevers. Blood cultures were sent. It came back positive for Gram positive cocci in clusters. What medication will you start = Vancomycin (Always mark Vancomycin if CVP line,Indwelling catheter or double lumen site Infection especially in hospitalized patient and that too in ICU or dialysis units. High prevalence of MRSA.) 193. Pt comes in to the emergency department with seizures. His BP is 90/60. His serum calcium level is 6, serum cortical level is 6 microg/dl , blood sugar level is 80 mg/dl. What is the diagnosis = Tetany 194. About Lignocaine what is true = Prevents influx of Na ions across membrane 195. INR is for = Oral anticoagulation therapy: ( For warfarin monitering INR >>> PT ) 196. A Person wid malignant melanoma. Risk factor = Xeroderma pigmentosum 197. Diabetic patient with DKA died due to = Mucormycosis 198. Urethra is narrowest at = External meatus 199. Married man had no children for few years and was diagnosd as sterile with azoospermia wat is investigatn of choice = Serum FSH and LH

Session 1

12

200. A man goes to a high altitude 02 delivery to tissues is maintained by = Hb conc is increased 201. Regarding indication of furosemide = Acute pulmonary edema 202. A patient was brought to the hospital after a roadside accident. On examination he was pale with low blood pressure and fracture of left 10th rib. Which organ is most likely to be injured = Spleen 203. Most common cause of premalignant change in mouth in adult = Lichen planus ( some colleagues saying chronic ulcer, none of the authentic books support that , and one more question often asked, that’s , what will beetle nut chewing cause and ans is submucosal fibrosis,) 205. Tumor in children with blue cells and high levels of catecholamines = Neuroblastoma 206. Ligament that prevents anterior dislocation of femur on tibia = Posterior cruciate (NOTE; if anterior dislocation of tibia on femure is mentioned then anterior cruciate ligament) 207. Cholangiocarcinoma caused by = Cholonorchis Sinensis 208. Trauma to left 10 rib causes damage to = Spleen 209. Farmer wid migratory lesion on foot = Cutaneous Larva migrans 210. Washed RBCs are used for prevention of = Hypersensitive Reaction 211. Eating raw or undercooked meat can cause infection by = Tapeworm 212. Man comes 1st time to Pakistan develops fever than coma, on investigation retics10%..Platelets wbcs ↓ = Falciparum 213. Correct about Aorta =. Gives renal arteries at L2 214. Scrotum drains to = Superficial Inguinal Nodes 215.Small muscles of hand are supplied by = T1 216. Pancreatitis(acute) will cause fluid accumulation in = Lesser sac 217. Deep ring present in = Fascia transversalis 218. Eversion is done by = Peroneus longus 219. True about Thoracic duct, drains into = At Confluence of left Subclavian and left Jugular 220. Gustatory sweating is caused by = Superficial Parotidectomy (Freys Syndrome produces flushing and sweating instead of salivation in response to taste of food after injury of AURICULOTEMPORAL which carries Parasympathetic secretomotor fibers to PAROTID gland and Sympathetic fibers to SWEAT gland. When the nerve is severed the fibers can regenerate along each

Session 1

13

pathway and innervate the wrong gland. It can occur after Parotid surgery and may be treated by cutting the tympanic plexus in middle ear- Ref BRS Anatomy) 221. Analgesic causing tachycardia in therapeutic dose = Pethidine 222. Following is a good analgesic but weak anaesthetic = NO 223. About Posterior Triangle of Neck what is true: = Contains subclavian , three trunks of brachial plexus 224. Herpes simplex virus transmitted by = .droplet 225. Pt came in with abdominal pain, his amylase was high, he was diagnosed with Pancreatitis. What is his omentum going to show = Fatty necrosis 227. Which carcinoma doesn’t show distant metastasis = Basal cell Carcinoma 228. Orofecal transmission = HEV 229. When tension in muscle increases near to rupture which does the relaxation = Golgi tendon organs 230. Golgi tendon organs respond to = Tension ( FOR BOTH 129 AND 130 QUES REMMEBER = TENSION AND TENDON i.e when tension on a muscle increases golgi tendon organs starts firing and relaxes it ) 231. Pacemaker of Heart = SA Node 232. Parasympathetic stimulation = Decreases HR 233. Sacral hiatus is formed due to = Failure of fusion of laminae of 4th and 5th sacral vertebrae (Sacral Hiatus is used to administer Caudal (Epidural) anesthesia) 234. Effect of drug on a body = Pharmacodynamics ( MNEMONIC = D FOR DRUG & D FOR DYNAMICS i.e. drug on the body is dynamic ) 235. Which is not blood supply of scalp = Maxillary artery 236. Hydroxyindol-acetic acid in urine = Carcinoid syndrome 237. Pt came with testicular mass. It was diagnosed as a yolk sac tumor. What is the follow up marker for this patient = Alpha-Fetoprotein 238. In the hospitals most common cause of wound infection = Staph aureus (E.coli 2nd most common) 239. Nerve loss in urogenital triangle which nerve involved = Pudendal Nerve 240. Neck of femur supplied by = Medial and lateral circumflex arteries 241. A man having recurrent bacterial infections with streptococci also having diarrhea which on examination showed giardiasis, further exam revealed normal T cells but ↓B cells levels of igM,igG and igA were given = Bruton’s agammaglobulinemia

Session 1

14

242. True about Pituitary = Drains to Dural sinuses 243. About Quadrate lobe, true is = Drains by left hepatic duct 245. Barr body is found in the following phase of the cell cycle = Interphase ( both DNA and chromosomes replicate (Karyotyping is done in metaphase stage to study for chromosomes ) 246. Old woman with swelling on external ear she also had gout wd -ve bifringent type crystals scenario was given = Monosodium urate 247. After Cholecystectomy surgeon places drains into = . Right Subhepatic Region 248. Pt having H/o cough cloudy yellow colour effusion fluid containing 98 % neutrophil and 2% lymphocyte the fluid was dull to percussion = Purulent exudate 249. Isoniazid causes side effects which can be prevented by taking which vitamin = Pyridoxine 250. Gas gangrene. Cause of death = Toxemia 251. Not a feature of malnutrition in adults = Thrombocytopenia 253. A tumor with all germ layers = Teratoma 254.Down’s syndrome = Trisomy 21 255. Ankylosing spondylitis is associated with = HLA B27 ( PAIR i.e. PSORIASIS , ANKYLOSING SPONDELYTIS , INFLAMATORY BOWL DISEASE RELATED ARTHIRTIS AND RIETER SYNDROME) 256. Female with fever for 10 days and chest pain that aggravates on lying down. Pain is originated at = Pericardium 257. 6 years old patient presents with rhinitis, edema and urticaria on external surfaces = Type I hypersensitivity 258. Immediate effects of severe hypotension = Oliguria 259. Transfusion reaction will occur if we transfuse a person with A+ve blood group, the following group = AB-ve 260. Epidural space is widest at =. L2 (5mm) 261. Commonest cause of hepatocellular carcinoma in developing countries = Both Hepatitis B & C (IF B plus C COLLECTIVELY not MENTIONED THEN CLICK ONLY Hep B) 262. Connections that connect cells; with movement of ions across = Gap junctions 263. HLA B27 associated disease = Reiter’s Syndrome ( if ankylosing spond present in options prefer that )

Session 1

15

264. Pregnant lady with bile duct obstruction. Diagnostic investigation = GGT (FOR NON PREG = CLICK ALP ) 265. About Trichomonas vaginalis what is correct = Is sexually transmitted 266. Anion gap = Diff between measured cation and measured anion 267. Occulomotor nerve supplies = ciliary ganglion 268. Infective endocarditis diagnostic investigation = Blood culture (blood culture for diagnosis & ASO titre FOR THE EVIDENCE OF INFECTION ) 269. Essential fatty acid = Linoleic acid (Also Linolenic Acid is among essentials) 270. Patient was brought to the ER unconscious. The doctor gave Naloxone but it was not working. What is the likely cause of toxicity = Phenobarbital 271. Patient lacerated the dorsal part of his wrist accidentally. He came in with difficulty to adduct the fingers and difficulty abducting the thumb. Which nerve was paralyzed = Ulnar Nerve 280. Pain from lower inferior segment of Diaphragm = Subcostal Nerves ( Phrenic supplies central tendon of diaphragm) 281. Cingulate gyrus blood supply = Anterior cerebral Artery 282. Dorsal Vein of penis drains into = Greater Saphenous Vein 283. Internal carotid bleeding stopped by compressing = C6 284. A patient has nodule on vocal cord. The most unlikely mechanism = Atrophy ( Hyperplasia = click it if asked the most likely ) 285. Edema of acute infection. Feature = Proteins >3gm (EXUDATE) 286. Vein not draining to great cerebral vein = Occipital 287. Extent to which a tissue gets disturbed by occlusion of its blood supply depends on = Rate of development of thrombosis 288. A 15 years old girl with fixed dilated pupils. Lesion is at = Oculomotor nerve 289. Axon arise from conical shape part of neuron cell body called = Axon hillocks 290. Peptic ulcer profusely bleeding from posterior wall of duodenum which artery involved = Gastroduodenal Artery 291. Proteins that escape into tissues from blood vessels = Reabsorbed by lymphatic system 292. Epithelium of kidney = endoderm + mesoderm 293. 8 month Baby boy with repeated rhinitis 2episodes of pneumonia in less then 2 month, low plasma B cells n decreased immunoglobulins = IgA deficiency 294. Which is not a 'basic tissue of body = Blood

Session 1

16

295. A 30 years old man had a road traffic accident and sustained fracture of femur. Two days later he developed sudden breathlessness. The most probable cause can be = Fat Embolism 296. Carpal Tunnel Syndrome = Wasting of thenar muscles 297. Which of the following statements about carcinogenesis is false = Papilloma viruses produce tumors in animals but not in humans 298. Max potassium absorption occurs in = PCT 299. Lung capacity increased in inspiration by 2/3 by = Diaphragm 300. Pituitary Gland Causing Bitmemporal Hemianopia.Where is the lesion = Optic Chiasma

Session 2

1.Hypermagnesemia = Neuromuscular depression ( hypo-Mg causes depressed deep tendon reflexes, muscle weakness, if dec acetyl choline release is present in options , click that) 2. Brain blood flow is regulated by = PCO2 3. Confidentiality can be breached = if patient authorize you 4. H202 in which organelle = peroxisomes 5. Cannot regenerate = Lens 6. 2nd arch derivative = Stylohyoid 7. Liquefactive necrosis = in brain 8.Golgi muscle tendon = causes muscles relaxation 9.interstitial lung disease with non-caseating granuloma & hilar nodes = Sarcoidosis 10.Most common genetic disorder = Multifactorial 11.Muscle of fright and Sadness = Platysma 12.Throid lymph Drainage = deep cervical lymph nodes 13.Temporal artery is related to = Auriculotemporal nerve 14.Acute Renal failure part affected = PCT 15.Shoulder pain referred from pleura = C345 16.Lt suprarenal drain = Lt renal vein 17.Councilman bodies seen = Apoptosis 18.Basic drug binds = alphaglycoprotein ( while acidic drugs bind albumin ) 19.Case control typical scenario = disease and not diseased 20.ICAM and VCAM = adhesion molecules 21.Symphysis pubis joint = Secondary cartilaginous joint ( all midline joints are secondary cartilaginous joints like symphisis menti, joint between vertebrae, symphysis pubic etc ) 22.Pyogenic peritonitis – Bacteroids 23.Dif f Malignant and Benign tumors = metastasis 24.Microscopic feature Malignant tumor = invasion 25.Goblet cells covert to Clara = Terminal bronchioles 26.No lymph nodules = Thymus

17

Session 2

18

27.Megalobalstic anemia invest = Intrinsic factor antibodies 28.Scenario Achondroplasia = Autosomal dominant 29.Pt lying on bed at room temp = Conduction 30.Decrease Gentamicin clearance in old age = Decrease renal func 31.Congenital cataract = Rubella 32.Virus act by =altering proteins synthesis 33.No valves seen in = SVC 34.Pt support in our culture = family 35.Fast pain fiber = A delta 36.Preganglionic nerve fiber = B fiber ( while post ganglionic are C fibers, and if aksed pure autonomic fibers, then prefer B fibers, and don’t confuse B fibers with A beta fibers) 37. Definitive diagnosis of TB = sputum AFB 38.Extraembryonic coelomic membrane = endoderm 39.Not isotonic to plasma = 5% dextrose water 40.Ventricular filling -= 3rd heart sound ( S1 = isovolumic contraction, S2 = isovol relaxation & S4 = atrial kick ) 41.Organ which utilizes glucose – brain ( if asked cell then click Rbcs) 42.Lower motor neuron lesion = flaccid paralysis 43.CNS myelination = Oligodendrocytes 44.Flight or fright Stimulation = Decrease release of bronchial secretions 45.Atrial fibrillation = Pulse deficit 46.Hot sunny day, sudden collapse = Excessive sweating 47.C1 vertebra feature = lacks vertebral body 48.Dec plasma proteins= Inc GFR 49.Cardiac reserve = Max % by which CO can be inc above normal 50.Osmosis depends= no of particles 51.5% dextrose in large amount given= ADH inhibition 52.Betel nut chewing causes= sub mucosal fibrosis 53.TB antibodies are = Cell bound 54.Regarding HIV = decreases helper T cells 55.Most imp response in shock as a whole = CNS ischemic response 56.In Iron def anemia in pregnacy, Increased = TIBC 57.Acute glomerunephritis proteinuria = Basement membrane defect 58.Motor unit = single motor nerve fiber innervates multiple muscle fibers 59. Extension of Hip and Flexion of Leg is by = Semitendinous 60.Long Thoraccic Nerve Supply = Serratus Anterior

Session 2

19

61. Correct Sequence of Protien Synthesis is = mRNA Cytoplasm,Ribosome,tRna,AminoAcids 62.All true regarding Breast Peau de orange appearance Except = Orange Brown Appearance is because of the Necrosis 63.Lady with leg Swelling from last 5 months with no history of fever and coug hbut cxr show bilateral peural effusion Derranged Lfts and Rt Heart Enlargement = Recurrent Thromboembolism 64.Estrogen, Progesteron and Testosteron acts Via = Transcription ( All steroid hormones act on Genes as they can easily cross membrane and need no cell surface receptors ) 65.Complication Associated with Trochanteric Fracture is = Pulmonary Emolism 66.GVE to palate Muscle is via = Dorsal Vagal Nucleus (G= Gen sensation ie not special like smell taste etc , V= Visceral ie Stomach ,Gut etc, E= Efferent ie nerves Exiting from CNS) 67.Max increase in ECF Volume due to = Infusion of Hypertonic Solution. 68.internal carotid a damage, bleeding, at what vertebral tubercle u will press to stop bleed = 6th cervical 69.Cysticercosis caused by = ingesting eggs of tenia solium 70.Which drug has more effect on cox2 than cox1 = celexocib ( if asked more selective cox2 , prefer celecoxib, if asked potent cax2 , prefer Meloxicam then ) 71.Suboccipital n supply which muscle = rectus capitis posterior 72.Regarding intercostobrchial nerve true is = 2nd intercostal sensory 73.Normal resonance are = right 2nd to 4th midclvicular line 74.In asian population most common cause of macrocytosis = alchol (if asked in our country then prefer liver disease) 75.RCA after giving marginal branch obstruction affect = AV node (BRS BCQ) 76.Penile urethra rupture = scrotum under colles fascia ( note the MNEMONIC ●PUS = Penile Urethra Scrotum/ Superficial perinial pouch ●BUS = Bulbar Urethra Superficial perinial pouch ●MUD = membranous urethra Deep pouch ) 77.Middle rectal artery branch of, = internal iliac 78.Chlamydial infection causing chlamydia psittacosis is,, = obligate intracellular 79.Scenario of granuloma with yellow pigment = Actinomyeces 80.Painful papule at labia = hsv II (HSV I is on lips Face , but now a days the rule is not followed strickly) 81.most perinium and pelvis nerve supplied by, = pudendal

Session 2

20

82.Which of the following Maintain flexor tone of arm mainly = rubrospinal ( note that rubroSpinal is for flexor tone while vestibulospinal tract is for extensor tone ) 83.Lower motor neuron lesion. = muscle wasting 84.Hemisetion with total spinal section differentaiad by = spinal shock 85.Which one is most common exocrine pancreas tumor = ductal adenocarcinoma 86.Delayed action of insulin = stimulate mRNA transcription for lipogenesis 87.Gas having maximum diffusion capacity in body fluids = CO2 (Max affinity is of Co and Maximum Diffusion capacity is of Co2) 88.Dec PO2 in blood. = Hypoventiltion 89.A person living on river side since 2yrs, having blindness, opthalmologist indentify which larva = onchcerosis 90.Antrum of stomach removed what will happen = dec compliance 91.Margins of TB ulcer = undermined 92.Ulcer due to Hypertension, = Martorell 93.What is the Mechanism of action of PTU = Inhibits synthesis of thyroid hormones 94.Right optic tract lesion causes = left homonymous hemianopia 95.Internal laryngeal nerve passes through = Thyroid n hyoid 96.Which is used for screening congenital abnormality = Estriol 97.Phenyl over alcohol anaesthesia = less painful (Alcohol is Very Potent and Excruciatingly Painful) 98.Patient developed hemolysis due toG6PdD deficiency,,What will happen Next = Self limiting 99.Liver Develops from = ventral mesentry proximal foregut 100.Saphenous nerve is terminal branch of = femoral 101.Adductor muscle hamstring muscle supplied by which nerve = tibial 102.Gracillis is supplied by the nerve = obturator nerve 103.Adductor brevis supplied by = ant branch of obturator nerve 104.Fracture of surgical neck of humerus, artery damaged, = post.circumflex humeral 105.Common bile duct parts = omental, retroduodenal, paraduodenal (Bile Duct has 3 parts only) 106.Inion is a landmark of, = external occipital prominence 107.Gastroduodenal artery branch of, = right hepatic artery ( actually its branch of common hepatic artery, but if it’s not in option the click right hepatic artery Ref RJ LAST) 108.Regarding tramadol true is = weak meu agonist

Session 2

21

109.Ketorolac analgesia most preferable = Non diabetic female undergoing cholecystectomy 110.Radiotherapy most sensitive to = craniopharyngioma (Seminoma >> Glial Glioma >> Craniopharyngioma) 111.Bilateral exopthlmos, antithyroid antibody present = graves 112.Which one drain into left brachiocephalic vein = inf thyroid ( while superior and middle thyroid veins drain into internal jagular veins) 113.A 64 year Old man with posterior cervical lympadenopathy which is firm andmobile. Hb 13-14 hct = 40 wbc 7230 plateletes 250000 no hepato splenomegaly seen biopsy showed numerous small monomophic lyphocytes, ,= .hodgkin lymphcyte predominant (cervical lymph nodes in the Neck is the most Common Site of HL) 114.Girl with h/o of gum bleeding, giant platelets , plt count 85k, = bernaud solier (note, B for Bernaud souleir and B for Big platlets ) 115.Terminal ileum removed what happens = large amount of water in feces 116.Protein content less in glomerular filtrate occur due to = pore size and negative charge 117.A 35 year old man bilateral crepts, on xray chest heart fills completely, mass in rt ventricle, what will b the main cause = myxoma 118.True regarding the Flexor Hallucis longus, = tendion can be felt immediately post to medial malleolus 119.Superficial parotid painless tumor, psudopoid growth, contain no capsule on histopatho = pleomorphic adenoma 120.A Very Fat Boy was eating burger & a very Large Bolus stuck after eating ..what will happen next, = secondary persiatlisis will be generated by intrinsic nervous system to remove it 121.When will u do dialysis with GFR Decreases to = 5 % 122.All drugs transported from Placenta by this mechanism = involves carriers ( also note that amino acids and fatty acids uses facilitated diffusion) 123.Regarding pituitary gland true is = vein drain directly in dural Venous sinuses 124.Patient with fluid loss now show metabolic acidosis e hypokalemia. most fluid loss will be through: = Colon 125.Anemia wid Blast Cells ins peripheral blood = Erythroblastosis fetalis 126.Closure of lips involve: or unable to close lips which muscle involved = Orbicularis oris 127.Superior Thyroid artery arise from = External Carotid Artery 28.Incubation period of which disease is longest = Infectious Mononucleosis 129.Age estimation at 5th to 12th week is done through: = crown rump length

Session 2

22

130.Shivering center is located in = Posterior hypothalamus 131.A 30 year old woman in azad kashmir was injured mildly in an earthquake that killed many people apart from her distant uncle. She would most probably be in = Situational crisis 132.Which ligament supports odontoid process of axis on atlas = Tranverse ligament 133.C7 Vertebrae has = Longest spine 134.Conversion of fibrinogen into fibrin what will b detected in blood = Fibrinopeptides are detected in Blood 135.Percentage of Plasma out of ECF = 30% 136.Newborn with a Erythroblastosis Fetalis having Blood group B positive what is best for management = B Negative 137.If a person is taking Protein in his diet which will be necessary = Riboflavin (also note that , Thiamine for Carbohydrates in Diet and biotin for fats metabolism) 138.Group of young men bathing n beech next day develop blisters on back shoulder limbs region cause is = Mast cell destruction 139.Regarding Renal Column contains= Interlobar artery ( though some friends states, that its collecting duct, not interlobar artery, I am with interlobar artery  #respect) 140.Deep to post digastric and near palatoglossus a structure runs obliquely upwards, = Lingual artery 141.H2O2 found in = peroxisome 142.ICAM and VCAM are for = Leucocyte adhesion 143.Breast atrophy in young female is due to = Estrogen and Progesterone 144.Virus in Human causes cancer via = Oncogen (Radiations via Protooncogens) 145.Essential amino acid = Phenylalanine 146.Spread of renal tumor will be through = Renal Vein 147.Retention of C02 = Respiratory failure 148.Regarding Gray matter foot processes = Protoplasmic Astrocytes 149.Cause of global blindness =Chlamydia Trochamatis 150.Which is normal HB = HbA 151.Thalassemia carrier percentage in Pakistan = 5% 152.Surgery for removal of ovary done which is damaged = Internal Iliac Artery 153.In respiratory Alkalosis renal compensation is = NaHC03 excretion 154.During mitosis chromosomes double in which phase = Interphase 155. Pain of ovarian tumor is refered to thigh by = Obturator nerve 156.Deep inguinal ring is present in = Transversalis fascia 157.17 Ketosteroid in urine seen maximally in =Normal male

Session 2

23

158.What is the relation of terminal CBD with head of pancreas= Embedded in it 159. After RTA patient becomes unconscious & on CT scan there is a hematoma which is compressing post central gyrus,s which area will be most affected = Sensory area 160.Lutenizing Hormone in males acts on = Leyding cells 161.True statement regarding Bone = Circumferential lamella arranged around central canal 162.Regarding Pneumotaxic Centre which statement is true = Inhibits Respiration 163.During exercise increased blood supply to muscles is via = Local metabolites 164.Spindle fibers will decrease discharge of impulses when = Muscle contracts 165.What is the Difference between systemic and pulmonary circulation = Low resistance in pulmonary circulation 167.MOA of steroids in Inflammation is = Inhibiting phospholipase. 168.there is a Trauma to upper limb and pt can’t feel sensation in little finger and paralysis of dorsal interossei which nerve is damaged = Ulner Nerve 169.which hormone causes extrahepatic protein catabolism and hepatic protein anabolism = Cortisol 170.Fragile x syndrome. = trinucleotide disorder 171.central chemoreceptors = change in PH of surrounding CSF 172.INTERnal carotid relation to external at beginning = Lateral 173.HIV female. Which one of the following carcinoma do occure = cervicaL carcinoma 174.post MI peripheral cyanosis scenerio cause = decrease cardia output 175.type 3 collagen (ehler danlos syndrome) = hypermobility of joints 176.pregnant lady ferrous sulphate taking. Decrease absorption of = Calcium 178.pertusis. = nasopharyngeal and bronchial secretion spread 179.allergic reaction with hypotension ( a big scenario is always there, but focus main points and ans is )= .i/m adrenaline 180.transport maximum not saturated scenario = Urate 181.carbon laden macrophages =pneumoconiosis 182.not natural immunity = plasminogen 183.REGARDING CSF = less proteins than plasma. 185.medical ethis. = moral code of conduct of doctr professional life 186.in portal HTN increases pressur in = inf mesenteric vein 187.pain transmitted from tip of finger = A delta fibers (sensations from the tips and lips are transmitted by A beta fibers, but those are not for pain, so don’t be confused) 188.defect of neural tube ,which marker increase. =alpha feto protein

Session 2

24

189.durng c section structure damage causing cramps in lumbar region = ureter 190.true about ureter = cross bifurcation of common iliac 191.aftr paralysis sensitive to pain damage in = thalamus (the so called thalamic pain syndrome, the person becomes sensitive even to normal stimuli) 192.in our society female death apart from obstertical death. = domestic violence 193. obstretical death comon coz in our country = hemorrhage 194.true relatd bladdr separatd frm rectum by = donovillier fascia 195.sperm stored aftr ejaculation in ... EPIDYDEMUS ( BRS ANATOMY STATES; “ Seminal vesicles do not store spermatozoa as once thought ; this is done by Epididymus , the ductus deference and its ampulla , if only one is asked prefer epidydemus) 196.a boy with testiculs swelling nd fever viral infection = mumps 197. hemophilia a gen located on = X chromosome 198. middle meningeal artery = passes thru foramin spinosum 199. labia majora vesicles lymph node = medial superficial inguinal node 200. true about ivc = comenses at L5 201.boy age 4 yr 4 days of fever,genraliz lyphadenopahy nd purpuric raash = infectious mononucleosis 202.G6pd deficiency = bite cell 203.suply of midgut form sympathetic = t5 – t12 ( sympathetic innervation , preganglionic from thoracic splachnic nerves t5 to t12 while postganglionic cell bodies ; superior mesenteric ganglion ref;kaplan anatomy) 204.infant with absense of limb cause is = amniotc bands 205.thyroid hormon. increas = Free fatty acids 206.which cozs decres productn of prostaglandin. = corticosteroids 207.about metaplasia = one epithelial changes into anothr normal epithelial 08.wafarin toxicityimediately treatd by = FFP ( if FFP and Vit-K both are given in one option, click that) 209.mifepriston through = progesteron antagonist ( Miferpristone stimulate uterine contractility ‘ antiprogestin’ also increases prostaglandins by inhinhibiting prostaglandin dehydrogenase) 210.goblet cell absent in = terminal bronchiole 211. preterm delivry = bactrial vaginosis 212.hydrocarbon cozs = bronchogenic carcinoma 213.adh and oxytocin arise in = hypothalamus 214.Alkaline phosphatase(obstructive jaundice)due to = stone 215.drug causing to inhibit breast ca metastsis to liver = tamoxiphen

Session 2

25

216.patient having pain on flexing thigh = pelvic appendix 217.viruses cause change = of protein alteration 218.radiaton cause change of = proto-oncogene 219.Premalignant lesion = Compound nevus. 220.Regarding middle meningeal artery most appropriate....Enters middle cranial fossa through foramen spinosum... 221.Minerals not found in or minerals with low bioavalibility = tubers 222.Medial boundary of ischiorectal fossa. = External anal sphincter (External anal sphincter and sloping fibers of levator ani forms medial border) 223.Boy with h/o trauma to perineal region, swelling in scrotum and ant. Abd wall but didnt go to thigh. Rupture of =Bulbar urethra 224.Turner syndrome = Short stature 225.Pseudo hermaphrodite With 46xy = Adrenogenital synd. 226.Abdominal aorta.= Gives renal arteries at L2 227.Stab to chest...air in pleural space will cause .= Lung to collapse and chest wall to sprung out. 228.Lateral thenar muscles spared but other short muscles of hand paralysed. Injury = Ulnar 229.axillary artery.relations = Post to pect minor 230.Infection ant. To pretracheal fascia spread to = Ant. Mediastinum 231. Primitve streak mesoderm migrates to anterior abdominal wall and around cloaca. A rupture of this mesoderm cranially will produce =.ectopic anal opening 232.Systemic fungal infection .drug given. = Amphotericin 233.Vitamin A =. Rhodopsin 234.S2 in comparison to s1 sound = Inc. Frequency ( S1 has inc dullness and duration than S2 ) 235.B/l recurrent laryngeal nerve cut. Muscle spared. = Cricothyroids. 236.Superficial cardiac plexus. = Contain both symp n parasympath 237.Gastric ulcer pain. = Greater splanchnic 238.Appendicitis pain radiates to umblicus thru = T10 239. H/o fall bladder incontinence unable to void urine. Spinal segment involved. =. S2s3s4. 240.Neck of fibula fracture. Dorsiflexion n eversion gone. =Common peroneal nerve 241.excessive eversion lig. Ruptured. = Deltoid. 242.Person standing. Venous return from legs mainly facilitated thru. = Contraction of sk. Muscles 243.Mediolateral episiotomy. Structure cut = Bulbospongiosus muscle 244.Regarding withdrawal reflex = Multisynaptic

Session 2

26

245.Tension in the muscle is limited by = golgi tendon organ 246.Alpha adrenergic causes.= Mydriasis 247.Due to stress of surgery inc = ACTH ( ACTH is stress hormone, but if cortisol is there in options, prefer that ) 248.In crf unlikely = Hypophosphatemia 249.Tcell receptor complex bind with. = MHC 250.Gluteus medius n minimus. = Abduct n medially rotate. ( whenever u see medial rotation of lower limb, close eye and click minmus and medius if there ) 251.Ciliary body = Pigment epi of retina is cont. With pigment epi of ciliary body. 252.Isthmus of thyroid lies in front of. = Rings 2 3 4 253.St elevation v4. Artery involved = LAD ( NOTE, that it’s the most commonly involved artery in CAD) 254.Lt.circumflex artery involved in mi. Old mi with new mi. Area involved.= Lt. Atrium n lt. Ventricle. ( left circumflex artery supplies both left atrium and ventricle) 255.Most commonly involved heart valve = Mitral valve 256.Dka pt. Died. Which fungal inf. = Mucurmycosis 257.Stroke death. Autopsy brain. Necrosis. = Liquefactive 258.Abdominal angina. Artery involved = SMA 259.50 yrs old. Hypertensive. Severe chest pain radiating to back. Died. On autopsy findings. = Medial necrosis of aorta 260.Median umblical lig is remnant of = Urachus. 261.Premature baby born most common finding is = PDA 262.Pulmonary arteries supply. = Alveoli 263.In adult head of femus blood supply. = Med n lat circumflex femoral art. 264.During lumbar puncture the structure punctured = Lig.flavum 265. Return of blood during exercise from =. Calf muscles 267.Testosterone main function = Inc muscle and bone growth 268.Sertoli cells function = androgen binding protein 269.Carb protein fat synthesis hormone = Insulin 270.Most common salivary gland tumor is of which salivary gland = Parotid 271.Cervical ca. Lymph nodes involved = Internal iliac ( if the option both internal and external iliac is present, prefer that) 272.Regarding rectum. = Also supplied by median sacral artery. (arises from posterior aspect of abdominal aorta just before bifurcation , descending in the front of sacrum and supply the posterior portion of rectum .Ref; BRS ANATOMY)

Session 2

27

273.Regarding anal canal = supplied by both sup n inf rectal arteries. 274.Coumadin (warfarin) question = inc coagulation time 275.Vit k not absorbed. Coagulation factor decreased .= Prothrombin 276. Vit k def. First n foremost coagulation factor depleted. = Protein c ( if protein C is not in options, prefer factor seven ) 277.Female presents with dvts . Says she has been taking some kind of pills. = Ocps. 278.10 days fever. Chest pain unrelated to respiration. = Myocardium 279.Breast outer quadrant = Pectoral (anterior ) group 280.Pregnant with jaudice. Went to remote village. To see her parents. Cause = Hep e ( note that ● commenset hpatitis is hepA anywhere, but if ● asked remote area, or epidemic or deadly in pregnancy prefer hep E then ) 281.Which ca rarely metastasizes = Basal cell carcinoma 282.BCC features. Most likely = Locally malignant 283.which ca spreads to large vessels = Clear cell ca of kidney ( even this carcinoma can reach the heart through inferior vena cava) 284. Cholangicarcinoma is caused by = Clonorchis sinensis. 285.Most common ca of epiphysis = Osteoblastoma ( if Giant cell tumor is present in options click that ) 286.limb growth will be affected if fracture passes thru = Epiphyseal plate 287. Acoustic neuroma. = Vestibular component of CN 8 288.Hypoglossal nerve injured. = Tongue deviated to same side 289.Intraarticular disc of Temporomandibular joint involved.. Muscle paralyzed. = Lat.pterygoid 290.Gastric lymphoma ass with = H pylori gastritis 291.Femoral arterial pulse = Mid inguinal point 292.Inguinal canal. = Roof Formed by conjoined tendon 293.In. Old age =. Venous Compliance decreases 294.Decrease in ESR is due increase in = Albumin. 295.Glycocalyx contains = Carbohydrate moiety. 296. in juxta glomerular nephron, sodium is actively reabsorbed electrically neutral from lumen of nephron in which segment of nephron = Proximal convoluted tubules ie PCT 297.Greatest clearance is of = PAH ( the order is PAH > Dietary potassium > inulin). 298.Rt. Testicular vein enter = IVC. 299.Tibial nerve severed. Which muscle will still flex the knee joint = Short head of biceps 300.oraganelle with double membrane = Nucleus

Session 3

28

Light can only shine through a broken vessel.Bear Grylls 1.Sacral plexus anterior division branch = obturator nerve (●Ant division nerves = tibial nerve L4-S3, obturator L2-L4 , Post division nerves = femoral L3-L4 ,common peroneal nerve L-S2 ,superior and inferior gluteal nerves Ref;KAPLAN anatomy) 2.In embryo lung is covered by mesothelium form which layer = Parietal pleura 3.Anterior to pancreas is = Lesser sac (note that; head of pancreas lies anterior to inferior vena cava and the neck/ body of pancreas lies anterior to aorta, while superior mesenteric artery is anterior to the uncinate process of pancreas ) 4.Most common dislocation of TMJ = Anterior (TM-Joint can dislocate only anteriorly) 5. A 60 years old man develops pain in calf muscles after walking hundred (100) yards pain is due to = Popliteal artery 6. Ulcer 2nd part of duodenum which artery will u ligate = Gastroduodenal artery 7.Extraembryonic coelomic membrane = Endoderm 8- For transfer of radicals from one group to another = transferase ( Note that ; oxidoreductases catalyze oxidation reduction reactions, ie electrons are transferred, while transferase transfers moieties from donor to recipient ) 9- For conversion of glucose-1-phosphate to glucose-6-phosphate require = isomerase (its actually phosphoGlucomutase , but if not present in options, click isomerase ) 10. Somatostatin act on beta cells by = paracrine action ( the GI Paracrines are, somatostatin and histamine, they defuse over a short distance to act on target cells ) 11.Gram negative bacteria causes infection of which system = Urinary tract 12. Per mint perfusion of 100g brain tissue per minute = 50-55ml (while brain glucose consumption is 5mg/100gm/min while oxygen consumption is 40ml/min) 13..Drug used to treat Endometriosis = medroxypregesteron acetate (click leuprolide if medroxy is not in options) 14.Toxoplasmosis is = A parasite 15. L1-L2 ends at = Inferior hypogastric plexus

Session 3

29

16.detrusor nerve supply = Parasympathetic pelvic nerves ( while pudendal nerve supplies external sphincter ) 17.which bronchus of lung is supplied by eparterial artery = right superior ( Right superior lobar bronchus is eparterial bronchus i-e branch of right main bronchus, it arises above the pulmonary artery due to this reason its name EParterial bronchus and all others arise below the pulmonary artery and are called HYParterial ) 18.correlation of two things by which test = student T test ( T for two, first aid ) 19. Sarcoma is differentiated from carcinoma = tissue of origin ( if this option is not present, prefer aggressiveness) 20. Narrowest point in pediatric airway = cricoid 21. If thalamus gets damaged, which is not affected = sense of olfaction 22.Superior colliculus receives which sensation = visual (Memorize as , the superior colliculus is above and eyes are above , while inferior colliculus is for auditory system and ear are lower to eyes) 23. Posterior communicating artery = passes above the occulomotor & connects ICA and PCA 24. Which muscle gets paralyzed/damaged in temporomandibular joint dislocation = lateral pterygoid 25.1st branch of abdominal aorta = inferior phrenic artery 26. dilatation of aortic arch will compress = left bronchus 27. axillary vein is formed by = basilic vein and vena commitantes of brachial artery (these form axillary vein at the lower border of teres major muscle) 28. injury to lateral hypothalamus will = decreases hunger 29. in open laparotomy, surgeon recognizes left kidney by = anterior relation with stomach,pancrease,descending colon,spleen,jejunum 30.linear growth pf bone is affected if following structure is fractured = epiphyseal plate 40. beta endorphins = hypothalamus 41. typical scenario of sickel cell anemia with abdominal pain jaundice = HbSS 42. a tall young girl(child) with history of secretion from breast.on microscospy of anterior pituitary, which cells are abundant = eosinophilic (GH and Prolactin are acidophiles ie chromophils and take eosinophilic dye ; Rest are basophils i-e cells other than these two varieties ) 43. which toxin is responsible for scarlet fever = erythrogenic toxin 44. highest sodium channel concentration = node of ranveir 45. Antibody receptors are = hypervariable regions of H and L chains

Session 3

30

46. Dicumarol side effect = inc bleeding time 47. mode of action of cephalosporin = inhibit transpeptidase 48. lithium is a toxic antipsychotic. when should it be stopped = coarse tremors ( katzung states ; lithium causes Fine tremors in therapeutic dose and coarse tremors in toxic dose) 49.Thyroid follicles that release hormone are derived from = Endoderm 50. steady pressure is detected by = ruffinis receptors 51. Best indicator of venous return = end diastolic volume 52. erythropoietin secretion is inhibited by = theophylline 53. Primigravida taking iron tablets, lots of fresh vegetables and juices .what deficiency she is at risk of developing = B12 54. child brought by mother,suffering from diarrhea.on examination,rectal prolapsed was found =. trichuria trichuris ( A hint is most of the time given that worm with small anterior end ) 55. what is more/high in venous blood as compare to arterial blood = PCV ( due to the chloride shift phenomenon in venous blood, water enters into the red blood cells, and cell swells, resulting in increased packed cell volume ) 56. in which condition atrial repolarization is shown on ECG = third degree complete heart block 57.child with hypocalcemia(or tetany),having some cardiac problems and viral infections = 22q-11 microdeletion (this is a scenario of DIGEORGE SYNDROME a Developmental failure of the third and fourth pharyngeal pouches, due to 22q11 microdeletion . Presents with T-cell deficiency due to lack of thymus and hypocalcemia due to lack of parathyroids anda abnormalities of heart ,great vessels, and face) 58. which valves are most commonly involved in rheumatic heart disease = mitral and aortic valves 59. which is more in dialyzing fluid as compare to plasma = glucose (composition of dialyzing fluid; ●Glucose & HCO3 are more than plasma, if both asked prefer glucose ●Na , K & Cl less than plasma ●Mg & Ca equal to plasma ●urea , creatinine, sulphate and zero,) 60. Hematocrit = rbcs+wbcs+platelets ( this ans is from Ganong BCQs while according to many others and even Dorland, hematocrit is just RBC,s and the buffy coat is formed by platlets and

Session 3

31

61. which antiemetic is preffered in pts of chemotherapy = odansterone 62. ranitidine is different from cemitidine as = it has less CNS toxicity 63. Lower limit of subarachnoid = S2 64.In spermiogenesis acrosome reaction by = lysosomes ( if asked , acrosome is formed by ??? then click Golgi ) 65.Receptor action via Phospholipase C = Vasopressin induced smooth muscle contraction 66.intertesticular temp less than rectal temp = 1-2c 67.Hb is 10, O2 saturation 90% ( convert 90% to 0.9 ), PaO2 = 60 , calculate O2 content = 13 ans (see solution below) Formula for oxygen content =(Hb g/dL ×1.34) × SaO2 + PaO2×0.003 oxygen content = (10 × 1.34) × 90% + 60 × 0.003 oxygen content = 13.4 × 0.9 + 0.18 oxygen content = 12.06 + 0.18 oxygen content = 12.24 i.e 13 68.Buccopharyngeal Membrane = Stomodium from Foregut (note that ; if asked Buccopharyngeal fascia then pharynx from foregut ) 69.Absolute contraindication of Propofol is = Hypersensitivity 70. Lung carcinoma fungiform growth. = Squamous cell carcinoma 71.sugar in urine will be detected if its concentration in blood becomes = 180mg (at plasma concentration less than 250 mg /dl all the filtered glucose is reabsorbed and excretion is zero, the plasma concentration at which glucose first appears in urine is approximately 250mg/dl Ref;BRS Physiology, while chatterje states, Renal t 72. Esophagus = Passes through left crus of diaphragm (Ref R.J.Last) (Snell says right crus and in paper left crus was not in options) 73. Aneurysm of aorta at diaphragmatic hiatus compresses = azygous vein and thoracic duct 74. Antacid decreasing gastric motility = Aluminum hydroxide (note that, aluminium causes constipation while Mg causes diarrhea ) 75. Vessel lying close to LAD , prone to injury during surgery is = great cardiac vein 76. Ascending reticular formation located in = floor of aqueduct of sylvius 77.Patient taking propranolol develops increased PR interval . what is the cause = Drug induced 1st degree heart block 78.Osmolarity of Human plasma = Similar to 0.9 % N/S 79. post ganglionic sympathetic fibers are present in = all spinal nerves

Session 3

32

( sympathetic outflow is from thoracolambar region, but post ganglionic fibers in all spinal nerves ) 80. For sodium neutralization = Cl ( while HCO3 for K neutraliztion) 81.That melatonin = secretion inc in dark (Pineal gland has a midline location above the quadrigeminal plate in the midbrain and is Site for melatonin production . Superior cervical sympathetic ganglia stimulates receptors on pinealocytes causes release of melatonin into spinal fluid and blood . Melatonin is Important in sleep/moods and circadian rhythms Released at night. Ref Goljan, plz don’t cofuse it with melanin) 82. Smoking is associated with = thromboangiitis obliterans ( it is also called BUERGER disease) 83. which of these is not synthesized by post ganglionic sympathetic fibers = L.Dopa 84.Bromocriptine reversed amenorrhea mechanism of action is = it acts as dopamine agonist 85. not a part of natural immunity = plasminogen 86. Factor 8 is synthesized by = Endothelial cells 87. what is the landmark for pudendal nerve block = ischial spine 88.Pt with HTN , LVH , Creatinine 0.8 , Urea inc, BSF 88mg% with urinary sugar ++ , Na 140 , K 2.5. Cause of HTN = Hyperaldosteronism ( the function of aldosterone is to reabsorb sodium and excrete hydrogen and potassium, that’s why in hyperaldosteronism, K is depleted and there is alkalosis, and Sodium level won,t fall because water is absorbed along with Na) 89.Which structure is embedded in temporal lobe = insula 90.Ureteric bud is formed as a part of = Mesonephric duct 91.Coccygeal ligament start at = L1 92. 40 years Old man from Sibi Baluchistan with fever and Lymphadenopathy with the H/O of sleeping on floor and often bite by flies. On Peripheral blood film Unicellular structure can be seen. Diagnosis is = Kalazar 93.Irreversible step in Glycolysis = Glucose to Fructose-6-Phosphate 94.Intracranial and extra cranial veins are joined by = Emissary Veins 95.Situation of SA node = Upper portion of Sulcus terminalis ( but BRS anatomy states ; SA node is a small mass of specialized cardiac muscle fibers that lies in the myocardium at the upper end of the crista terminalis near the opening of the svc in the right atrium.) 96.Thromocytopenia in Children due to Acute Viral infection is because of = Antibodies against platelets

Session 3

33

97.Esophagous narrowest at = Cricopharyngus 98.Upper and lower limits of cerebral blood flow autoregulation = 50150ml/min (The ranges mentioned by Francis physiology is 60 to 160 which is almost near to 50 to 150) 99.In some children Dorsalis pedis Artery is missing congenitally. Then dorsum of foot will be supplied by which artery = Peroneal artery 100.Itching will stimulate .= C fibers 101.A Child got blow at elbow and elbow joint displaced. And loss of sensation over little finger. Structure involved = Ulnar collateral ligament 102.-A 35 years old man die suddenly. Autopsy finding nill. Toxicology shows cocaine and its metabolite in blood. Cause of death = Contraction band 103.Ganglion found at T7 vertebra = Thoracocervical 104.Edrophonium most common adverse effect = Nausea 105.Damage to first lumbar vertebra will lead to = . Conus Medullaris Syndrome 106.In feamle neck of bladder lie = Lie above urogenital diaphragm 107. Mechanoreceptors of Warm = Long receptive field 108.Cuneatus and Gracillis damage will result in = Asterognosia 109.Articular surface has which cartilage = Hyaline 110.Elastic cartilage is found at = Epiglottis 111.Preganglionic Presympathetic fibers = B type ( note that post ganglionic are C fiber and if asked what are pure autonomic fibers again click B fibers, plz don’t confuse B fibers with Beta fibers ) 112.PICA damage, blood supply of which part will be affected = dorslateral of medulla 113.Chromatid align at equatorial plate = Metaphase ( also chromosal study is carried out in metaphase, and if asked replication of any organelle click interphase) 114.Which Cells have telomerase = Germ cells 115.Perineal area innervation was asked = S2-4 116.Most drugs are metablized in = Liver 117.Death due Clostridium botilinum = Paralysis of respiratory muscle 118. True about Supernumary cervical Rib = Compress brachial plexus and subclavian vessels 119.A female with blurred vision etc case of Multiple Sclerosis = Demyelination of neurons 120.Node of Ranvier = Lack of Myelin sheath 121. tuberculin test (Negative) = Taking immunosuppressive drugs

Session 3

34

122.Bupivacain first sign of over doze = ringingin ears (if asked about serious adverse effect, prefer arrythmias) 123.A child suffering from 104F fever, and has been on anti malarials .His lfts are deranged wd inc unconjugated bilirubin.What is the most probable diagnosis = blackwaterfever ( if in the ques , high temperature is not mentioned then prefer G6PD deficiency ) 124.about sertoli cells = release androgen inhibiting factor( so called inhibin) 125.Baby Diper gets yellow due to = biliary atreasia (if biliary atreasia is not in options , prefer Gilbert then) 126.Medial quadrant of breast drain into = Internal thoracic nodes 127.Superior sagittal sinus pressure in standing position = – 10 i-e ―minus ten ― (Ganong physiology) 128.Aspiration of hydrocele,needle does not pierce = tunica albugenia 129.shoulder pain reffered from pleura = c3,c4,c5 130.which stucture passes through esophageal hiatus = Vagus trunk (right vagus) 131.adrenocortical deficiency causes = Hyponatremia ( the job of mineralocorticoids is to reabsorb sodium, thas how the deficiency of mineralocorticoids may result in hyponatremia ) 132.malarial parasite transfer in which form in man = Sporozite 133. Most common congenital anomaly of head and neck = Cleft plate 134.most commonly fructure of bone after 60 yrs = Neck of femure 135.Most common Inheritance pattern = Multifactorial 136.fructure humate bone = Ulnar nerve (A fall on the outstretched hand may fracture the hook of hemate which may damage the ulnar nerve as it passes into the hand) 137. ivu kidney more prominent = Calyces 138.spinal cord ends at = lower border of L1... 139.epidural space contains. = Venous plexues 140.Regarding cephalic vein = Lie between deltoid and pectoralis major muscle 141. A 10 yrx boy repeated visits to opd with pallor frontal bossy = thalasemia major 142.renal vein lies posterior to = Superior mesenteric vein 143..lumber triangle anteriorly bound by posterior border of = Serratus anterior 144.Mononucleosis. Stucture most likely to be damaged even on minor trauma = spleen 145.Barium swallow Feathery appearance in left abdomen = Jejunum

Session 3

35

146.Damage to middle cranial fossa. No lacrimation. Damage to = Ciliary ganglion 147.Which of the following is not isotonic to plasma = 5% D/w ( 5% dextrose water is almost same as pure water) 148.Highest Pco2 in = Pulmonary artery 149.Somatostatin acts on beta islet cells = Paracrine 50.Rabbit disease = tularemia 151.Nerve asked, loss of sensation in lower teeth, chin and lower lip = Inferior Alveolar nerve 152.Retromandibular vein is formed by junction of Maxillary vein and = Superficial temporal vein 153.Alpha-2-agonist used in ICU for sedation = Dexmedetomidine ( Dexmedetomidine is alpha-2-agonist like clonidine and is most often used in ICU setting for light to moderate sedation, the reason to prefer is , it has analgesic and hypnotic properties and is not associated with significant respiratory depression ) 154.1st lumbrical = attaches to radial side of index finger ( Details; The first lumbrical of the hand arises from the anteriolateral surface( ie radial side ) of the tendon of flexor digotrum profundus to the index finger) 155.Chondrocyte change into osteocyte = thyroxin 156. Muscle damage in difficult delivery = levator ani Muscle 157. Sperm will called abnormal > cerebral hemorrhage) 19. Cause of delay in healing – Infections ( infection is the most common cause of delayed wound healing )

Session 5

60

20. PaO2 decreased, PCO2 increased, hydrogen ion increased; manifestation –Hypoventilation. (look at CO2 which is increased that means person is not ventilating properly to remove it out that is HYPOVENTELLAION) 22. Person with tachycardia, and heat intolerance with low level of TSH, on giving TRH; level of TSH and thyroid hormones increases. Diagnosis – Hyperthyroidism with thyroid problem. 23. Origin of oxytocin and ADH – Hypothalamus. 24. Difference between systemic and pulmonary circulation - Low resistance in pulmonary circulation. 25. Mean systemic filling pressure is regulated by - Venous return. 26. Systolic pressure is directly related to which one of the following – Renin. 27. ADH responds to – Osmolarity. 28. Osmoreceptors are receptors for – ADH. 29. Right border of heart on X-ray also visible a part of –SVC. 30. In MI sensitive cardio marker – Tropinin T. ( IF asked in 1st hour or Enzyme in stem then prefer CKMB) 31. Diabetic nephropathy investigation - Urine albumin. 32. Young boy with generalized edema and proteinuria - Lesion of basement membrane. 33. Lesion of parasympathetic system affects mostly - GI muscles. 34. Decussating of medial lemniscuses - Internal Arcuate Fibers 35. S2 sound heard on - Closure of aortic and pulmonary valve. 36. A patient with history hemorrhage (trauma) receives a bag stored for 2 weeks mainly contains – RBCs. 37. Due to inspiration what decreases – Volume of atrial pulse. (as during inspiration there is minimal blood in the left ventricle that dec blood pressure) 38. Important buffer of blood - HCO3-. 39. Max increase in ECF due to infusion of - Hypertonic NaCl. 40. Auscultation of tricuspid valve best heard at – Right lower end of the body of sternum. ( this is CPSP favorite most question ) 41. GVE vagus nerve for preganglionic fiber arises from – Dorsal nucleus. ( it‘s the parasympathetic nucleus of vagus to the viscera that why its supply is so called , GENERAL VISCERAL EFFERENT ) 42. Tip of scapula at level – T7.

Session 5

61

43. Diabetic female after abdominal surgery; dyspnea and cough - Pulmonary embolism. 44. MCC of pulmonary embolism – DVT. ( NOTE ; if asked the most common site of DVT then Popletial vein if asked most common source for pulmonary embolism then FEMORAL) 45. Typhoid fever 1st week test – Blood culture 46. Typhoid fever 2nd week test – Blood culture and Widal test. 47. Typical feature of falciparum - Black water fever. 48. Alcoholic patient with deranged LFTs; on biopsy – Mallory bodies. 49. Potassium is mainly regulated by - Aldosterone. 50. Natural self-defense against tumors – Apoptosis. 51. P53 gene absent results in - cell survival. 52. Pain mediator - Bradykinins. 53. Metaplasia – Functional change in cells. (metaplasia is replacement of one adult cell to type by other , and is mostly due to CHRONIC IRRITATION , it‘s a REVERSIBLE condition and most common examples are BARRET ESOPHAGUS AND SQUAMOUS METAPLASIA OF LUNG DUE TO SMOKING) 54. Female with infection of HPV, comes after 2 years, Pap smear shows prominent nucleoli and increased nucleus size – Dysplasia. (Whenever in scenario u see word , change ins SHAPE, SIZE , ORIENTATION, ARCHITECTURE, OR INC NUCLES ISZE, always pick Dysplasia , its also a reversible condition) 55. Gas exchange occur - Simple squamous epithelial layer. 56. Patient with granulomatous disease, biopsy done. Microscopic finding that suggests TB – Caseous necrosis ( if asked the feature of granuloma then click epithelioid cells, not otherwise, if asked what will u do next after u see caseous necrosis then click AFB STAINING , if no clue is given the click AFB STAIN) 57. The spindle fibers will decrease in discharge of impulses when - Muscle contracts (NOTE; the job of spindle is to contract muscle, it means when the muscle contracts , its job is done and it DEC FIRING ) 58. Diagnosis for leprosy, initial investigation - Nasal scrapping. 59. Benign neoplasm – Adenoma. 60. 3 germ layers tumor – Teratoma. 61. When adrenalin release from medulla, causes vasodilation by acting on Beta 2 adrenergic receptors. ( ALPHA 1 = contraction , Beta 2= dilation )

Session 5

62

62. Increased GFR and increased plasma flow occur due to - Dilation of afferent arteriole. 63. Characteristic of cerebellar lesion – Dysmetria 64. Emax (maximum effect) of a drug depends on – Efficacy (cram it, is highly repeated) 65. Study in which every person of a population has equal chances of being selected – Random sampling. 66. Amniocentesis is done - After 14th weeks. 67. 1st response against acute inflammation in tissue – Macrophages. 68. In dark granules containing cells; IgE attaches to – Basophils. 69. Opsonization - C3b. ( also Fc portion of IgG is opsonin, BUT C3b is more potent) 70. Exudate - more than 3g of proteins. 71. About active transport of drug all are true except - All drugs pass via active transport. 72. Pulmonary artery supply to – Alveoli. ( this is the evergreen question among repeated ones) 73. Muscles of back innervated by - Dorsal rami. 74. In young boy dyspnea produced on lying - Retrosternal goiter. 75. Most important cause of bronchogenic cancer – Smoking. 76. Edema caused by - Increased hydrostatic pressure. 77. Edema caused by - Lymphatic blockage. 78. BP 210/180mmHg and creatinine 8% damaged part – Juxtaglomerular apparatus. ( this que is is mostly repeated In same way, so remember if very high BP an raised creatinine and in options there is JG APPARATUS. Don‘t click anything else) 79. Female with blood group A, have 2 children; one with O and other with AB, blood group of father is – B. 80. Genetically true hermaphrodite XXY ( This is one of the most controversial MCQ, and according to Goljan xx is correct ans that‘s in 50% of cases while other combinations make only little percentage, but cpsp key makes XXY a better option) 81. DNA replication occur in – Interphase. ( DNA & CHROMOSOMES replication occur in interphase) 82. Glycogenolysis caused by deficiency of which hormone – Insulin 83. Investigation for liver amoebic abscess – Serology.

Session 5

63

84. Surgery of submandibular gland; nerve damaged - marginal mandibular branch of facial. (This branch of facial nerve runs superficially and is mostly damaged during surgery) 85. On posterior surface of oblique and transversalis fascia - Arcuate line. 86. Aspirin overdose causes – tennitus. (Aspirin …. if asked GI disturbances = click normal dose, if tinnitus then= OVERDOSE) 87. Cardiac output measurement via thermodilution – Temperature change downstream with CO. ( NOTE ; if thermo dilution not mentioned then Ficks method) 88. Father with defective gene on one autosomal chromosome, develops disease later in his life; chances of getting disease in children – Half of the children will be affected. 89. If left circumflex artery occluded - Infarction of left atrium and left ventricle. 90. MCC of multiple fractures in adult – Osteoporosis. 91. Patient with fracture of many bones and low BP immediate treatment Volume replacement. 92. Most common fracture of long bone - Tibia. 93. Collagen fibers – Eosin stain 94. Gamma efferent supply to – Intrafusal muscle spindles. ( memorize by heart, its highly repeated) 95. Micturition – Self generating. 96. Hallmark of HIV – Proliferation of virus in T cell ( if asked about AIDS, THEN progressive immune deficiency). 97. 1g protein gives energy – 4 kcalories. 98. Isotonic and isometric contraction difference is that isotonic contraction – comsumes more phosphate bond. ( memorize as such , highly repeated) 99. Autonomic nervous system – parasympathetic increase salivary secretion. 100. Protrusion of mandible – Lateral pterygoid. 101. After feeding the baby, diarrhea develops – Gastrocolic reflex. 102. Both sides of 5th posterior intercostal spaces is supplied by - Thoracic aorta. 103. 2 years old child with medially rotated arm lesion in - Upper trunk. 104. Female with uterus cancer which travels to labia majora following which route - Round ligament.

Session 5

64

105. Pain in epigastrium due to ulcer, transmitted by - Greater splanchnic nerve. 106. Vesicular lesion on the tip of nose and eyelid - Ophthalmic nerve. 107. Patient can‘t abduct till 30 degree, which muscle is responsible – Supraspinatus. 108. Nerve supply to plantar surface of foot - Posterior tibial. 109. Medial arcute ligament – Psoas major muscle. 110. Bitemporal hemianopia due to lesion of - Central part of optic chiasm. 111. Ventral spinothalamic tract - Coarse touch and pressure. 112. Dorsal column – Proprioception. 113. Vibration and touch via - White matter of posterior spinal cord. 114. Function of notochord - Induction of ectodermal neural plate to produce neural tube. 115. Midbrain reticular system – Unusual stimulus causes arousal. 116. Most radiosensitive tumor – Glioma. 117. Hormone increased during sleep as compare to at day wakefulness – Cortisol. 118. In Trigeminal neuralgia anesthesia given to block trigeminal ganglion at which site – Middle cranial fossa. 119. Patient with intentional tremors and facial paralysis lesion at – Cerebellopontine angle. 120. Opioids produces vomiting by acting on - Chemo trigger zone in medulla. 121. Patient working in mine with tuberculin test positive. Cough, fever, weight loss with hilar lymphadenopathy – Silicosis. ( silicosis = predispose to TB that‘s why tuberculin positive) 122. Male working in construct factory with pleural plaques – Asbestosis. ( asbestos predispose to bronchogenic carcinoma and mesothelioma ) 123. Neoplastic cells with continuous mitosis due to persistence of – Telomerase. 124. During exercise venous return is due to - Contraction of calf muscles. 125. During exercise increased blood supply to muscles - Local metabolites. 126. Action of steroids as anti-inflammatory is due to - Inhibiting phospholipase. 127. Intermediate filaments (keratin) staining with immunofluorescence to diagnose – Carcinoma. ( note that if there is mentioned carcinoma then look for keratin , and if meshenchymal tissue tumor then VIMENTIN) 128. VPL nucleus of thalamus when damaged cause – somatosensory loss of contralateral body.

Session 5

65

(REMEMBER , Ventral posterolateral nucleus is for somatosensory sensation from contralateral body while ventral posteromedial ie VPM is for sensation from face ) 129. Thiazide diuretic cause – Hypokalemia. 130. Man living on side of a river, got blindness from nematode; organism Onchocerca volvulus. ( highly repeated) 131. MCC of HTN – Idiopathic. 132. Adult female with breast atrophy due to decrease in – Estrogen plus progesterone. 133. Female with hirsutism, galactorrhea, infrequent heavy menstrual cycle. U/S both ovaries with cystic lesions and LH:FSH :: 3:1 – PCOS 134. Female with hot flushes - Increase FSH and LH. 135. Injection of atropine will cause - Dryness of mouth. 136. Female with severe loss of vibration, Investigation of choice - Stomach biopsy. (this scenario is about vitamin B12 deficiency where there is sub-acute combine degeneration of spinal cord resulting in loss of vibration & position sense , the most common cause of vitB12 deficiency pernicious anemia, for which stomach biopsy is indicated ) 137. Child with yellow sclera and colored urine, best Investigation - Bilirubin and ALT 138. Female with weakness and lethargy with RBCs 5.5 diameter and very thin red line of RBC in peripheral smear - Microcytic hypochromic anemia. 139. Monitoring of Hep B – Serum glutamate-pyruvate transaminase. 140. Female with reactive anti HBe and reactive total anti HBc along with nonreactive HB IgM and non-reactive HepBs Ag - Acute recovery phase. 141. Patient with obstruction jaundice; now bleeding cause - Vit K deficiency. 142. Investigation of Hashimoto thyroiditis – Antimicrosomal and antithyroglobulin antibodies. 143. Cimetidine interaction with warfarin causes - Decrease hepatic clearance of warfarin. 144. Patient using some anti-coagulant, how will you know that anticoagulant is warfarin because it is being monitored by – PT ( remember the oral anticoagulant means warfarin and to moniter warfarin INR > PT, so if INR is there then click it if not then PT) 145. BP drops from 100mmHg to 70mmHg – Decrease in carotid sinus nerve activity.

Session 5

66

146. Erythropoietin is secreted by – Mesangial cells. ( erythropoietin is secreted by peritubular capillaries, but in CPSP exams, it is secreted by mesangial cells  #respect) 147. Patient with all symptoms of Cushing along with increased ACTH cause Cushing disease. ( simple difference between CUSHING disease and syndrome is , that in cushing diseae ACTH will be high ) 148. Child with machinery like murmur – PDA. 149. PaO2 is 100% in - Left to right shunt. 150. Kallmann syndrome – Defect in Arcuate nucleus. 151. Diplamotoylphosphatidyl choline + carbohydrate + proteins is a content of – Surfactant. 152. In asthma = Decrease FEV1. 153. Splenectomy is most useful in - Hereditary spherocytosis. 154. Findings in glycogen storage diseases - Hepatomegaly and hypoglycemia. 155. Corynebacterium diphtheria produces – Exotoxins. 156. Common specie of Corynebacterium affecting human is – Diphtheriae. 157. Typical symptom of food intoxication by Clostridium botulinum - Flaccid paralysis. 158. CSF with increased proteins and polymorphs with decrease glucose the drug of choice = Ceftriaxone. 159. About the pharmacology of Neiserria gonorrhea = Pencilin G. 160. Major histocompatibility is related to – HLA. 161. Kidney is transplanted, gets cyanosed in 10 minutes, type of reaction Antibody mediated. ( In case of hyper acute rejection, there are preformed antibodies, which reacts within minutes,) 162. A child with Agammaglobulinemia is prone to - Pyogenic infections. 163. Diagnostic criteria for sarcoma - Increased vascularity. 164. Soldier coming down from height, now with discoloration of fingers, dyspnea - Secondary polycythemia. 165. Vitamin D acts on intestinal mucosa via - expression of genes. 166. Release of neurotransmitter due to - Influx of calcium in presynaptic fiber. 167. Calcium influx via slow channels in ventricle cells causes - Ventricular ejection. 168. Increased ‗a‘ wave in JVP due to - Tricuspid stenosis. 169. Person with HR 80/min, on ECG RR interval would be - 0.75 sec. 170. Class III Antiarrhythmic – Amiodarone. 171. Increased ASO titer - Rheumatic fever.

Session 5

67

172. In hypertrophic cardiomyopathy – Myocytes disarray. 173. 5 month of pregnancy, lady with thyrotoxicosis, drug of choice – PTU. 74. 30 years old smoker, psychotic, vomiting of fluid with visible peristaltic movements on abdomen cause -Pyloric stenosis. ( note that the stomach that you hear ( auscultation give splash sound) , you can see ( visible peristalsis ) and you can feel ( olive like mass on palpation ) is pyloric stenosis ) 75. Myenteric system of parasympathetic is potentiated by – Ach. 176. Pethidine is preferred over Morphine – Less addictive ( Another FACT frequently asked about pethidine is , that it causes TACHYCARDIA at therapeutic dose , WHILE morphine causes release of Histamine) 177. Philadelphia chromosome – CML. 178. Male with fever upper lobe consolidation, fever from last 3, productive cough, AST 135 and ALT 111, alkaline phosphatase normal, bilirubin 10, he died on autopsy weight of liver increase and with fat droplets seen on hepatocytes and Mallory bodies , Diagnosis - Chronic alcoholic disease. 179. Alpha 2 macroglobulin inhibit – Trypsin. 180. Male with all symptoms of LV failure and pulmonary edema without any previous illness and BP 125/85mmHg, on X-ray cardiomegaly; cause of heart failure – Bicuspid aortic stenosed valve (If the history of alcohol intake is given, prefer alcoholic cardiomyopathy) 181. Highest potential difference in the wall of stomach is when – Aspirin ingestion. 182. Calculate Alveolar ventilation where Tidal volume is 500ml/respiration. RR 10 - 3500ml/min (Alv ventilation = TIDAL VOL – DEAD SPACE 500 – 150 * 10=350*10 = 3500 ) 183. Calculate Clearance where Urinary flow 20 ml/min, Concentration of substance in urine 10 and Plasma 10 – 20ml/min. ( clearance = UV/P i-e 20 *10/10 = 200/10 = 20) 184. Excessive eversion of foot damaged which structure - Rupture of deltoid ligament. ( if asked about inversion then talofabular ligament , inversion sprains are more common than eversions) 185. About descending aorta . give branches to pericardium lung etc 186. Counterpart of Mullerin tubercle - Seminal colliculus.

Session 5

68

187. Histology of cornea - Stratified squamous non-keratinized. 188. Left shift of HbO2 curve - CO poisoning. 189. Humans are intermediate host in - Hydatid cyst. 190. Clot retraction is mediated by – Thrombosthenin 191. In late pregnancy the uterus is most sensitive to – Oxytocin. ( while for the initiation of labour fetal cortisol is important ) 192. About bicep brachialis muscle – attaches to supraglenoid tubercle of scapula. ( cram it, its highly repeated ) 193. Hyper-resonance is mostly heard at – T2-T4 of right midclavicular line. ( because in this region there is only lung, while on left side there is heart and right side below this ,is liver, so this area is always hyper-resonant) 194. Conus medullaris terminates in neonates at the level of – L3 195. Cimetidine is used preoperatively – Dec acid secretion from stomach. 196. Nursing mother with herpes labialis, drug of choice – Acyclovir. 197. PO4 increased, Calcium decreased and PTH increased – Chronic Renal Failure. ( remember if PHOSPATE IS INC and Ca is dec THEN FIRST EXCLUDE CHRONIC RENAL FAILURE) 198. Labetalol acts on – Alpha and Beta receptors. 199. Women after delivery, Platelet: 70,000, aPTT normal, PT raised – DIC. ( Also note the point that DIC is initiated by THROMBOPLASTIN ) 200. Between umbilicus and suprapubic rectus sheath behind at its lower limit forms – Arcuate line 201.which of the following cranial nerves have parasympathetic nuclei = III, VII, IX,X ( 3. 7 . 9 & 10 ) 202.What is hamartoma = totaly benign (Hemartoma is just jumbled up normal tissue and totally benign) 203.Adrenergic Receptors of Adrenal medulla : cholinergic ( specifically cholinergic nicotinic) 204.Coronary Arteries - always run n in the respective AV septal depression, 205.Scalenus Anterior Muscle - phrenic nerve passes anterior to it 206.loss of sensation of nose, cheek, maxilla, zygoma : maxillary division of trigeminal nerve (if tip of nose mentioned then opthalmic division of trigeminal nerve , the most fav question of cpsp) 207.Which of the following have sub capsular afferent sinuses : lymph nodes (Whenever you fine sub capsular afferent in stem , first look for lymph nodes in options)

Session 5

69

208.which of the following hormone increases content of skeletal muscle Proteins—testosterone 209.An Ovum is surrounded by corona radiata which is formed by : granulosa Cells 210.Patient underwent colostomy after colon resesction for colonic cancer, best way to monitor for recurrenc; CEA levels 211.Growth harmone : is a polypeptide a second option is often there in paper,acts indirectly via IGF, if u find it,click it 212.Man with hx of repeated URTI, hx of passage of fowl smelling stools for 10 years , he would have deficiency of which of following : , vit D ( passage of foul smelling stools indicate loss of digestion and absorption of bowel fats resulting in deficiency of fat soluble vitamins, vit D is fat soluble, ) 213.neurogenic shock is different from other types of shock because of : loss of vasomotor tone 214.bleeding time increased : VWbrand's disease ( in vw disease , both bleeding time and APTT are prolonged ) 215.left kidney grossly identified from right kidney by : arrangement of structures in hilum 216.which of the following have highest incidence of bronchogenic carcinoma = smoking 217. A man with third degree burns have increased risk of developing : Contracture 218.Which of the following arches over root of right lung : azygous vein (● structure arches over left lung root = aorta cz on left there is aorta, ●structure arches over right lung root = azygous vein as its on right, ●structure anterior to the root of both lungs = phrenic nerve,ie phrenic in the front ●and posterior to the roots of both lungs = vagus nerve) 219.Stored blood has following characteristics ; increased hemolysis 220. Fibrocartillage : present b/w intervertebral discs 221.breaking bad news : should be told to patient in a formal session exclusively (if option another option is present click that, and that option is , PRESENT INFORMATION IN A CRISPY AND ACCEPTABLE WAY ) 222.for efficient patient doctor relationship best way to start conversation by ;;, asking about his name, age, address 223.regarding thyroid : active follicles are filled with colloid 224.QRS complex is slightly prior to : ventricular systole

Session 5

70

(i-e electrical activity before mechanical activity) 225.which of the following predispose to malignancy : proto-oncogenes ( while in case of virus click oncogenes ) 226. Myxoid degeneration will occur in which : mitral valve prolapse 227.which of the follow prevents flow of substances in intercellular space : tight Junctions 228.most common cause of fatty liver in our country ; fatty food ( if protein malnutrition present in options then go for it ) 229.Aspirin prevents formation of; thromboxane A2 230. Anthrax is 90 % associated with : cutaneous lesions 231.Hairy Leukoplakia is associated with : AIDS 232.boundaries of Digastric Triangle : anterior and posterior bellies of digastric muscle and ramus of mandible 233.blood supply of foregut is primarily derived from : celiac trunk 234.man encountered fracture of femur in RTA, died after 5 days, cause of Death = Fat embolism ( if pregnant lady is mentioned and death Is immediate then amniotic fluidembolism , if time is in days then fat embolism even in pregnant lady will be the ans) 235.follicular cells of thyroid are derived from = endoderm (follicular cells of thyroid are simple cuboidal epithelium and their embryologic origin is from a median endodermal mass in the region of the tongue , in contrast to the parafolliclar c cells that arise from the 4th pharyngeal pouch) 236.Neutrophillia will be seen in : after an acute attack of MI ( like all other inflammatory conditions, in MI as well, in day 1 to 3 , neutrophils stay in there, while from day 4 to day 7, macrophages dominate the site ) 237.Hepatic pedicle is formed by : portal vein, common hepatic duct, hepatic artery ( means portal triad) 238. adult male of long hight, well developed , presented with complain of infertility what could be the cause : klinefilter syndrome 239. Nor-Epinephrine and Serotonin are degraded by : MAO, 240. Most potent Response in massive haemorrhage is : CNS ischemic Response 241. In nephrotic syndrome oncotic pressure decreases due to : hypoalbuminemia 242.Isthmus of thyroid gland is present in front of : 2nd,3rd, 4th tracheal Rings 243.Alternative of schwan cells in CNS : oligodandrocytes

Session 5

71

244.Regarding Rectus sheeth tendoneous intersections : upper most is located near xyphoid process 245.Regarding Hymen : lined by stratified squamous 246.Histology of Vagina : str. squamous epithelium with abundant elastic tissue in submucosa, highly vascular, smooth muscles around ( whenever u see ABUNDANT ELASTIC TISSUE in the stem, look first for histology of vagina ) 247.Histology of Ureter : small lumen tubular str. lined by transitional epithelium, with complete muscular coat 248.Mother contracted german measles during pregnancy , fetus is on risk of developing : congenital Catract, 249.Risk of Congenital catract by Rubella virus can be prevented by : vaccination of girls of child bearing age 250.Which of the following can be palpated via lateral fornix of Vagina : ureter 251.tapping of patellar tendon causes : contraction of quadriceps muscle 252.Pts of different races are coming at emergency , 50% are whites, 20% blacks, 20 % Asians, best way to depict there percentages is : pie chart 253.During later stages of pregnancy , Ripening of Cervix is caused by : Hydration of Collagen 254.Simple columnar Epithelium with Goblet Cells and submucosal lumphoid tissue : Ileium ( note that submucosal glands of Brenner are present in duodenum while lymphoid payer pateches are present in lamina propria and submucosa of ileum ) 255.Patient presented with coagulopathy with dead fetus in uterus most imp. step of initial managment : Evacuation of Uterus 256.A lady is receiving radiotherapy for CA-Cervix, which of the following will be seen on histology : karyorehexis and karyolysis ( chemo and radio both cause apoptosis, that how histology will show the features of apoptosis) 257.acute effect of high dose Radiation : endarteritis obliterans 258.Oliguria is production of urine in 24 hours : less than 500ml, 259.Patient with normal amount of semen production but azospermia : defect in sertoli cells 260.GnRH regulates secretion of : FSH and LH 261.IN preganancy mother feels respiratory distress b/c of = dec.in total lung capacity 262.young female with hx of multiple recurrent abortions : anti-phopholipid syndrome 263.Hypophysectomy will effect primarily : principle cells of thyroid

Session 5

72

264.A young female patient presented with pain RIF with hx of amenorrhea for 6 weeks, pregnancy test is positive : ectopic pregnancy (Acute appendicitis if leucocyte count, nausea and fever mentioned ) 265.Urogenital diaphragm lateraly attaches to : ischiopubic rami 266.Pregnant patient presented at 3rd month of pregnancy with Hb 12.5 g/dl, later on presented on 9th month with HB 10 g/dl normocytic, normochromic, cause of anemia : normal physiological change 267.local anaesthetic MOA of ionized substances : cause blockade of Nachannels 268.Neuromuscular junction motor end plate : contains acetylcholinestrase 269.Patient presented with Hx dyspepsia, anemia, pale hands, spooning of nails, stool test showed OVA : Ancylostoma Duodenale ( note that , ascarisias casuse GI obstruction and lung problems, trichuriasis cause dysentery and rectal prolapse, entrobius vermicularis cause pruritis ani, and onchoceercasis cause river blindness) 270. A 23weeks pregnant lady died in RTA type of death is : Fortitious death, 271. While doing appendectomy via incision at Mc'burney's point artery is damaged : deep circumflex Iliac artery ( if asked about , which STRUCTURE is damanged then , ans should be illeohypogastric nerve) 272.Elderly patients are prone to develop drug toxicities b/c of : dec. renal Excretion 273.Patient having menstrual cycle of 32 days will ovulate on : 18th day (the last 14 days of the cycle are fixed, any change in the length only occurs in the days before it, which may either reduce of prolong the total duration so to get the ans ; 32-14 =18) 274.Which of the following will cause bradycardia : occulocardiac reflex, 275.neoplasm having rete ridges , malignant infilterating epithelial cells : Verrocous carcinoma 276.In pregnancy there is : hypoalbuminemia 277.Lymphatic drainage of skin around anus : medial group of superficial horizontal lymph nodes 278.Bite cells will be seen in : G6PD def, 279.Inferior layer of urogenital Diaphragm : perineal membrane 280.which of the following is benign tumour = warthin tumor 281.septum primum and septum secundum fuse to form septum which separtes : common atria into right and left atria 282.At the end of a marathon race there is = Increased glucagon and low insulin

Session 5

73

283.A 7 years old child having generalized edema Investigation of choice is = urine albumin (& for diabetic nephropathy the investigation of choice is = urine albumin ) 284.Apex of the heart = 8 cm from median plane apex of heart 285.Alpha and gamma motor neurons = Supplied by UMN 286.trisomy 21 =Robertsonian translocation 287.agressiv ca = Melanoma 288.regarding malignanct charcter = metastasis 289.polyphagia in diabetes ocures due to = inc blood glucose 290.edema of kidney origin is due to = albuminuria and Na retention 291.subacute endocarditis = viridins ( viridns causes sub-acute and damaged valves endocarditis, and is on the top over all, while staph causes acute and in healthy valves, mainly in iv drug abusers, though in new editions, Staph is on the top overall, but for cpsp. Prefer viridins  #respect) 292. Pus contains = dead neutrophils 293.at beginning of 5th week = heart definitive formed ( though in 4th week, four limbs and four chambers of heart are formed and at 4th week primordial germ cells migrate into undifferentiated gonads, but if asked definitive heart then click beginning of 5th week) 294.contraction of pupillary muscle = parasympathetic 295.hormone causing gluconeogenesis = cortisol (gluconeogenesis ; prefer cortisol over Glucagon, but for Glycogenolysis , prefer Glucagon, though it can cause both) 296. primordial germ cells derived from = Ectoderm (Primordial germ cells have been isolated from the epiblast at posterior end of primitive streak in 2nd week while germ layers are formed in 3rd week by the process of gastrulation, hence primordial cells can‘t be considered, the derivative of any of these 3 layers, its believed that, epiblast is the future ectoderm that how, the correct answer is epiblast, if not in options click ECTODERM. Earlier it was believed that PGC‘s are derived from endoderm of yolk sac) 297.ureteric bud derived from = Caudal end of mesonephric duct 298.lenght of Right main bronchus = 2.5cm (repeated so many times) 299.anterior (costosternal surface) of heart is formed by = right ventricle 300. The most important component of shock is = tissue hypoxia

Session 6

74

1.Veins of thorax drains into = azygous drains into superior vena cava ( azygous vein, after arching over the root of right lung drains into superior vena cava behind the sternal angle, also note that there are no valves in superior vena cava, brachiocephalic veins and azygous system of veins) 3.Regarding primary peristalsis = begins when bolus enters the esophagus 4.Cardiac transplantation rejection, the cells involved are = T cell (CD8 CYTOTOXIC T CELLS : Recognize antigens in association with class I MHC Proteins Kill virus-infected, neoplastic, and donor graft cells via release of perforins and granzymes. While in chronic rejection, CD4 T CELLS are involved, as stated “Pathogenesis of chronic rejection, Most likely due to a chronic delayed hypersensitivity reaction involving CD4 T cells” ) 5.Tumor cells in the blood , most commonly diagnosed by = tumor marker 6.P53 mutation = cell survival ( P53, brings about cell apoptosis i-e death, means if its mutated, cell will suffer no death and will survive) 7.Liver abscess= serology is done for diagnosis 8.Biopsy taken from posterior triangle = damage to the spinal part of accessory nerve (spinal part of accessory nerve lies in posterior triangle, and is frequently damaged during biopsies or drainage of abscess from this are, not that the loss of accessory nerve result in paralysis of trapezius, and as a consequence loss of overhead abduction and inability to comb hair) 9.Alpha receptors = mainly in the proximal urethra 10.Parasympathetic = inc salivation ( Salivary secretions are inc by both symp and parasymp , but parasym effect is more dominant one.) 11.Receptors causing nor-epinephrine mediated dilation=β2 12.Which muscle causes jaw protrusion = Lateral pterygoid 13.Pancreatic tissue in stomach is = choristoma 14.Sickle cell disease protein affected is = beta globin 15.Iron deficiency anemia = inc TIBC (note that. The most sensitive test for iron deficiency anemia is Serum ferritn )

Session 6

75

16.At normal condition, both are equal = venous return and pulmonary blood flow ( the reason is that all the blood entering in right side of the heart should be pumped to lungs for oxygenation) 17.Difference between pulmonary and systemic circulation = low resistance in pulmonary circulation 18.Long term memory immunoglobulin is = Ig G ( the initial Ig is always IgM , that’s followed by IgG ) 19.Profofol absolute contraindication =Hypersensitivity to profofol ( profol appears safe in porphyria , note that , barbiturates contraindicated in porphyria ) 20.GVE comes out of = dorsal nucleus of vagus ( its also known as posterior nucleus of vagus and is the main parasympathetic outflow of vagus ) 21. Which of the fallowing structures doesnot receive innervations from branches of pudendal = Posterior Fornix of Vagina 22. Sructure attached to Lumbar Verebra = Anterior Longitudinal ligament 23. Right horizontal semicicular canal stimulated = Head tilt right 24. Athetosis is due to defect in = Globus pallidus (according to Guyton and hall) 25. Opsonization occurs with the = Help of c3b 27. Propulsive movement of intestine effected due to lesion of = Aurbach plexus 28. Main class associated wid antigen presenting cells = MHC2 29. About small intestine true is = small intestine 6-8 meters long 30. Lysosome has secretion against bacterial iron because they contain = Hydrolases 31. Blood supply of heart is regulated by = Local metabolites 32. Post synaptic sympathatic mainly secretes = Nor epinephrine 33. True abt vertebral column = Cervical nerves originate abovecorresponding vertebra (Except C8 cervical nerve which is below C7 vertebrae , and all Spinal nerves also originate below the corresponding vertebrae) 34. Confidence interval = Mean n its standard error of mean 35. Positive Predictve valve is affected by = Prevalence of disese (Remember PPP = Positive Predictive value is affected by Prevalence) 36. In PPD induration u will find = T cells ( if T-cell & macrophages are present in options prefer that) 37. Which of the following drugs is contraindicated in a patient with raised intracranial pressure = Ketamine (mnemonic = KRANIUM VS KETAMINE)

Session 6

76

38. If circumflex branch of left coronary artery is blocked which area will b effected = Posterior surface of left ventricle 39. Draining to Sup Mesenteric nodes = Jejunum 40. Diabetic pt came to ER insulin given what change will happen in blood = increased pH ( as dec PH is due to acidosis by ketone bodies and insulin when given, will dec ketone bodies and will inc ph) 41. Potassium will be lost mostly frm which segment following dietary irregularities = distal tubule 42. Main diagnostic feature of DIC = presence of D dimers 43. A man wd his gut removed some weeks ago now showing ↑ gastric motility wid diarrhea, cause = Osmotic+secretory diarrhea 44. CO poisoning, methemoglobulinemia, anemia dont result in hyperpnoea, this is because = Po2 remains normal (Anemia- Pa02 and Sa02 – Normal MethHB and CO Poisoning - Pa02 is normal but , Sa02 DECREASED) 45. Ductus deferens end in = ejaculatory duct 46. Sjogren associated with = RA 47. Ovarian pain related to = Obturator 48.Antibiotic given , patients PT and APTT deranged what will u give = vit k 49. Subdural hematoma due to = rupture of cerebral vein (note that; ●Epidural = Middle Meningeal Artery=Lens shaped Biconvex hematoma ●Subarachnoid = Cerebral arteries and Vein (Berry Aneurysm in Circle of Villis) ●Subdural = Superficial Bridging cerebral Veins =Cresecent hematoma) 50. Enzyme in saliva that prevents iron utilalisation by bacteria = Lactoferrin 51. Fio2 max dose which does not cause fetolental adrenoplasia = 0.6 (Safe limit is 0.6 or less than that) 52. Gardener right side chest pain 3.5 cacific on rt lower lobe = Aspergilosis 53. Recurrent renal stone associated with which amino acid abnormality = Lysine 54. Nerve supply of Lattisimus dorsi = Thoracodrosal Nerve 55. Nerve supply of Rhomboids = Dorsal scapular nerve 56. Nerve supply of Sartorius = Femoral Nerve

Session 6

77

58. Vitamin produced by intestinal bacteria = Vitamin K (Newborns have no gut flora and are deficient with vit k , that why soon after birth vitk is administered to the neonates ) 59. End product of purine metabolism = Uric acid 60. Arrangement of renal segments: Renal-> Interlobar->Arcuate>Interlobular->Afferent Arteriole (According to SEMBULINGAM ; Renal arteries directly arises from abdominal aorta and enter the kidney through the hilus, while passing through the sunis , it divides into many segmental arteries which subdivide into interlobar arteries, each interlobar artery passes between medullary pyramids and at the base of pyramid forms arcuate artery, each arcuate artery gives rise to inter lobular arteries , from each interlobular artery numerous afferent arterioles arise) 61. Cervical CA metastasize to = Internal and External LN 62. Prader Willi Syndrome = Hyperphagia and Hypergonadism 63. A patient comes to you in your clinic with lab findings of RBS 198 G/dl and FBS 122 G/dl; what is your diagnosis = Impaired glucose (●Fasting >110 200 with Classical Symptoms and FBS must be > 126) 64. Which of the following have direct relationship = pulmonary blood flow and venous return (Factor That control Cardiac Output also control Pulmonary Circulation-Guyton) 65. In hypoxic vasoconstriction; which of the following is decreased = arterial pO2 66.Screening is done = to detect disease at its early stage 67. In athletes, which of the following is increased even in resting state = Stroke volume ( if not in options the prefer cardiac output) 68. Muscle of quiet inspiration = diaphragm 69. If a foreign body is inhaled; which of the following lung lobes it will go to = Rt inferior lobe 70. Intracellular ANIONS = Proteins 71. Type of junction present in smooth muscles = gap junction 72. Type of junction present in cardiac muscle = gap junction 73. A man moving in an elevator; which of these is working = saccule ( note that ●angular acceleration = semi circular canals,

Session 6

78

● linear horizontal ie in a car = utricle and ●linear vertical acceleration ie in an elevater = saccule) 74. Which of the following has normal anion gap = Diarrhea 75. Which of the following is diagnostic of tuberculous granuloma = epitheliod cells 76. A lady with 26 wks gestation has uterine height of 24 wks. On ultrasound there is no fetus and snowstorm appearance. Which of the following probably associated with it? = karyotype 46 XX 77. A soldier comes with heavy bleeding. The ideal fluid replacement would be = whole blood 78.Structure accompanying superior temporal artery =auriculotemporal nerve 79.Relation of terminal CBD with head of pancreas is = embedded in head of pancrease 80. Genotype of a true hermaphrodite is= XXY ( in goljan xx is given , while some of the colleagues saying that XXY IS CPSP KEY, though I couldn’t find xxy in any of the cpsp demos, still seniors agree to it, so ans is xxy , follow cpsp blindly no matter what Goljan says  #respect) 81. Calcitonin is secreted by which tumor = Thyroid 82. Angiotensin 2 is responsible for =systemic vasoconstriction of venules 83. Most common skin malignancy in pt with HIV = Kaposi sarcoma 84. Left gastroepiploic artery is branch of = Splenic artery 85. Organelle that binds carbohydrates with protein enclose them in vesicles n release = GOLGI bodies 86. Child with recurrent infection, which of the following physiological antibody is missing on peripheral smear = igA 87. Plasma membrane thickness is = 7-11 (7.5 - 10nm is Range- Guyton) 88. Lady with ventricular rate of 40/m most likely = complete heart block 89. Iron binds to = transferrin 90. Highest carbohydrates is in = wheat (71gm CHO) 91. Resonance of the lungs can b auscultated from back of the chest till = 8th rib (Demarcation Line is 3rd-9th Intercostal Space) 92. Flexion of knee and extension of hip is the action of = semitendinosus (Also Biceps femoris, Semitendinosus, Semimembranosus - Posterior Compartment Thigh muscles--> Tibial Nerve) 93. Which of the following bones has no muscular attachments = talus 94. Lymph drainage from outer nipple is to which group of lymph nodes = anterior axillary

Session 6

79

95. Dorsal scapular nerve supplies which of the following muscle = levator scapulae (Dorsal scapular Nerve also supplies Rhomboids) 96. Which of the following is a pivot joint = atlanto axial joint 98. Motor supply of pharynx is by = nucleus ambiguous (Nucleus Solitarius: Sensory & Ambigous: Motor) 99. Sacroiliac joint = plain variety of synovial joint 100.Mediator of pain = bradykinin ( Bradykinin > PGE2) 101.RT coronary artery = supplies S.A node 102.Most imp hormone involve in gluconeogenesis = cortisol 103.A pt presented with rt iliac fossa pain. he is diagnosed to have acute appendicitis on flexing his rt thigh inward n medially he cries with pain. What is the type of appendicitis he has = pelvic 105.Most potent chemotactic factor is = C5a 106.what is the most appropiate ans = C5a is a potent chemotactic agent 107.Low serum complement seen in which condition = SLE 108.Pt presented with photosensitivity,rash n joint pain.which is the daignostic in this case = anti DS antibodies 109.Which of the following has highest cholesterol content = LDL 110.Which drug does not cause gynaecomastia = Griseofulvin (Some colleagues saying androgen, but remember that gynecomastia is one of the adverse effect of androgen, and beyond all explanations, griseofulvin is cpsp key) 111.Which is the best option in identification of Turner's syndrome = karyotyping 112.End product of purine metabolism is = uric acid 113.Erythropoitin is secreted fro = Mesengial cells 114.ESR is decreased with = albumin is increased 115.RBC'are having = glycolytic enzyme activity 116.Commonest Cause of PDA = permaturity 117.Most common Cause of metaplasia is = chronic irritation 118.What is the differentiating point of hyperplasia fron hypertrophy = Increase in size n no of cell 119.Dysplasia is seen mainly seen in = epithelia 120.What is the differentiating feature of hyperplasia from benign tumo = increase in no of cell 121.What is the most true among following = rRNA is most abundant & involve as ribosomes on endoplasmic reticulum 122. Deficiency of THAIMINE(B1)causes all except =subacute degenaration of

Session 6

80

spinal cord 123.AIDS is associated with all of the following except = leukemia 124.VITAMIN involve in collagen synthesis is = ascorbic acid 125.Neural tube defects occur due to deficiency of = folic acid 126.Iron is stored in the form of = ferritin 127.CO2 is tranported in lungs through = simple diffusion 128.HPV is associated with = CA cervix 129.Optic groove appears on left side of forebrain on day = 22 ( memorize by mnemonic, 2 eyes and 22 days ) 130.Germen measles causes which abnormality. What is the most appropriate answer = congenital cataract 131.A newborn with anterior abdominal wall swelling and umbilical cord attached to it.what is the condition called = omphalocele 132.PT has exophthalmos & his T3 n T4 are increased.what is the option for treatment of exophthalmos = drugs blocking the lymphocytic infiltration 133.Treatment of diabetes in pregnant lady is = insulin 134.side effect of streptomycin = impairment of hearing 135.Which of the following is opportunistic organism = klebsella 136.Psudomembranous colitis is caused by = Cl.difficile 137.About aspargillosis which is not correct = is caused by long term antibiotic use. 138.Pt has a history of infection 4 weeks back.now he developed same pain again.what type of cell will present at this time of disease = neutrophils 139.What is the drug of choice for DVT in ist trimester of pregnancy = heparin 140.Type 2 hypersensitivity reaction includes = erythroblastis fetalis 141.Transfusion reaction will not occur in a pt if we transfuse the=group A with O ( as O is universal donor ) 142.Which of the following does not cross placenta easily = Ig M 145. Which drug is used in the treatment of hirsutism = antiandrogens 146.Most common organism involved in infection caused by IUCD = actinomycosis 147.ECF differ from ICF in = high conc. of Na & Ca ( if only volume is mentioned in the options the click that one) 148.diffrence of smooth n skeletal muscle = Ca-Calmodulin complex is present in smooth muscles 149.In partial mole,karyotype is = 69XXY (46XX is complete mole ) 150.If carbohydrate is separated from proteins what will be the fate =

Session 6

81

aggregation & precipitation 151.Lysozome contains = hydrolases 152. Following is Autosomal Dominant Disorder= Hereditary Spherocytosis 153-Capsule of liver in known as = Glissons 154-Most common site of thyroglossal cyst is: = Infrahyoid 155. Cancer is Least Related To = Traumatic injury 156. What Age The Head Of Fetus Become Exactly Half The Length At CRL = 3rd Month 57. Not Included In Blood Coagulation Profile = Anti thromboin 3 58. Starling Law of the heart =Explains increase in Cardiac Output that occurs when venous return is increased. 159. Scenario Of Fissure. Pain Due To Which Nerve = Inferior Rectal 160. Pendular knee jerk is the result of = UMN lesion (Pendular knee Jerk cerebellum >> UMN ) 161. Medial side of arm is supplied by =. Medial cord 162. Bare area of liver is limited by= Coronary ligament 163. During a surgery a surgeon nicks hepatoduodenal ligament. Which structure is likely to be damaged = Bile duct 164. Capsule of left kidney prevents the spread of infection to = Other kidney 165. Which one is secreted after sweating = Increase ADH 166. Baby born edematous aftr 2 days shows signs of renal failure on examination rounded mass in pelvic area on autopsy cystic mass is seen. Diagosis = failure of union of mesonephros n ureteric bud 167. B lymphocytes are associated with = CD 19 & 20 168. Which of the following ulcer is caused by synergistic infection by microaerophilic non hemolytic streptococci and aerobic hemolytic staphylococci = .Meleney's ulcer 189.testies drain into : para-aortic lymph nodes ( as gonadal vessels arise from abdominal aorta, that why drainage is towards para aortic lymphnodes ) 190.Best way to diagnose intraamniotic infection is by : detection IL-6, 191.During exercise increased blood supply to muscles is due to = Local metabolites 192.True regarding male reproductive system = Testosterone produced by leydig cells of testes. 193.ovarian cancer first metastasize to = par aortic lymph nodes. 194.structure passing through pelvic inlet is = ureter 195.Mullerian duct counterpart = rete testis cords

Session 6

82

196.common iliac arteries = crossed by ureter at their bifurcation 197.Ileococcygeal ligament located at level of = L2 198.which of the following is most likely correct regarding lesser pelvic in female = has an outlet bounded by ischeopubic rami and sacrotuberous ligaments 199.secondary oocyte indergoes the 2nd reduction devision = after fertilization200.in which of the following conditions ocps contraindicated = migraine 201.after 5 years of transplant surgery there is an increase incidence of : = lymphoproliferative disorders 202.longest incubation period = HIV 203.primary oocyte surrounded by zona pellucida and zona granullosa and fluid filled spaces is most likely = secondry follicle 204.female 28 weeks gestation,, presents with poly hydroamnios what is the problem with fetus = duedenal atresia 205.which of the following are boundaries of pelvic inlet = pubic crest 206.female with cervical dysplasia, , which test will be 95% reliable = pap smear 07.Pulmonary blood flow hindered by = Pulmonary embolism 208.ligament which held uterus in anteverted position and can be palpated on rectal examination = uterosacral ligament 209.Female with arthritis, rashes,, rynouds phenomenon, which is "specifiic" test = anti Ds antibodies (note that if asked about sensitive, then ANA is more sensitive) 210.sensitive marker of myocardial damage = Troponin T 211.WHICH of the following is associated with adult polycystic kidney disease = cerebral hemorrhage ( if sub arachnoid heamorrhages is present in options, prefer that) 212.isotonic and isometric cintraction. isotonic contraction is different by = consume more phosphate bonds 213.anemic hypoxia is caused by = methehemoglobinemia 214. inc in both renal blood flow d GFR = dilatation of afferent arterioles 215.chronic renal failure cause of anemia = Erythropoieten dec production 216.gamma motor neurons are present at the = end of intrafusal fibers 217.lady with major abdominal surgery develops dyspnea = Pulmonary embolism 218.before giving sodium valproate = LFT,s ( as the most dangerous adverse effect of valproic acid is fulminant hepatitis ) 219.function of hcg = maintains corpus luteum

Session 6

83

(Prevents from involution and any of the options can be asked so memorize both) 220.gold standard for breat cancer = Mamography 221.53 years old lady admitted in comatose stated, blood sugar 400, +4 sugar in urine,, no ketone bodies and albuminuria = DM2 222.Philtrum of upper lip = Tw0 Medial nasal processes 223.intermittent pain of small intestine is felt = Umblical region 224.after 15 days of c- section due to obstructive labour.. pt developed urinary incontinence. what is the most likely cause = Vesico vaginal fistula 225.breast lobes = separated by interlobar stroma 226.how good is a test at correctly diagnosi diagnosing people with the condition. Feature qualifies = Sensitivity 227.pO2 is lower in arterial blood than pulmonary blood because-- diffusion to occur 228.lady with back ach on right side,neck of femur fractured due to slipage in washroom.Blood calcium is raised and calcium appeared in urine,, diagnosis = hyperparathyroidism 229.enzymes are specific for male and female hormones production,,, enzyme only for female hormones production = aromatase 230.male pt with severe hemorrhage,presented in unconscious state,, transfused with 2 weeks old blood ,Blood mainly contain = Rbcs 231.cells which contain sense granules and have igE antibodies on their cell membrane are = basophills 232.current bone density is assessed b = dexa Scan female presented with some epithelial changes nd hpv +ve.Again presents and pap smear shows large cells with large nucleus.Diagnosis = dysplasia ( note ; Cervical dysplasia, HPV infection > IUCD, while most common cause of cervical metaplasia is Multiparity > IUCD) 233.Tachyphylaxis = Diminish unresponsiveness adapted quickly (in simple words, rapidly developing tolerance to a drug is called tachyphylaxis) 234.Powerful vasoconstrictor = Cocaine 235.True about pudendal nerve = Supplies muscles of perineum 236.GnRh is not inhibited by = Carcinoma 237.Ureter is supplied by all except = Inferior mesenteric arteries 238.Myleperoxidase is an enzyme present in = Neutrophil

Session 6

84

239.Fourty ears lady with 2 year history of pruritis ,for the last 6 months having xanthoma, hepatospleenomegaly, which investigation will confirm Diagnosis = Antimitochondrial antibody (a scenario of primary biliary cirrhosis) 238.Milk presents in the breast during pregnancy with normal prolactin Level but no lactation it is due to inhibitory effect of = Progesteron (actually estrogen and progesterone both result in the development of breast during pregnancy but at the same time they also inhibit lactation, if in options only one is asked then PROGESTERON should be preferred ) 239.Inferior orbital branch is a terminal branch of = Maxillary artery 240.Amniotic fluid embolism = During labor and postpartum (CMDT states , “though uncommon still amniotic fluid embolism is one of the complication of labor and immediate postpartum period” ) 241.Nerves/muscle not leaving pelvis thru greater sciatic foramen = Obturator 242.Parturition is caused by or started = Fetal cortisol (fetal cortisol increases oxytocin receptors on uterus there by increasing its sensitivity secondly it enhances the enzymatic degradation of Progesteron, resulting in termination of pregnancy) 243.Cardiac output maximum in = After delivery ( though not an easy concept; but let’s simplify, ●● ●after delivery a lot of blood enters into maternal circulation, that inc blood volume and BP, secondly we know that increasing the number of capillaries decreases blood pressure, ie via parallel resistance mechanism, after delivery, placenta is lost decreasing number of capillaries and thus BP shoots up, that’s why most common cause of death after delivery is cerebral hemorrhage, .. Ref Francis physiology) 244.Pars distalis = supplied from superior hypophyseal artery 245.burkits lymphoma is caused by EBV because = Antibodies to viral capsid are present in all individuals with burkitt lymphoma 246.40 years lady with post coital bleeding and inter menstrual bleeding on examination hypertrophic cervix, first investigation to do = Cervical smear 247.Inscisionall hernia 1st structure taking from within outwards = Transversalis fascia 248.Succinylcholine = Actions is prolonged due to the deficiency of psuedocholinestrase ( because this drug is inactivated by enzyme pseudocholinestrase) 249.If 28 day cycle on day 21 progesterone will be = 10ng (During Mid leutal phase progestrone should be more than 10 ng) 250.Insulin dependent uptake of Glucose into =skeletal muscle

Session 6

85

(Sembulingam states that; usually glucose is transported into the cells by NaGlucose co-transporter , but most the cells have another transport protein called glucose transporter ie GLUT, among these GLUT-4 is insulin sensitive and it Is present in large amount in muscle fibers and adipose cells and via GLUT-4 transporter, insulin pushes glucose into two cells, ieadipose and skeletal muscles ) 251. transpyloric plane is located at = lower border of L1 ( structures lie at the level of trasnpyloric plane = pylorus of stomach , fundus of gall bladder, Hilum of kidney, first part of duodenum , origin of superior mesenteric artery the tips of 9th costal cartilages, and lower end of spinal cord if lower border of l1 is not there click upper border of L2) 252.Pyruvate intermediate between = Glucose and acetyl coA ( Glycolsis ends up giving 2 molecules of pyruvate ,which after conversion into actyl CoA and enters krebs cycle ) 253.Pretracheal infection will be transferred to =Anterior mediastinum 254.Spinal anesthesia is given to a patient, there is sudden loss of sensation due to = loss of vasomotor tone 255.Insulin increases by = GIP ( Gastrin, CCK , GIP , secretin and GLP-1 all inc insulin secretion, but GIP AND GLP-1 are the most potent ) 256.Basal ganglia is involved in = Initiating movement 257.Hypokalemia on ECG is represented by = U wave 258.Regarding the end arteries = Present more in the vital organs ( ●End artries then central retinal artery > splenic artery ● if asked functional end artry then click heart ) 259.Female in labour, station 2 on PV, ischial spines inverted, arch less triangular, which type pelvis is this = Anthropoid 260.Blood leaving placenta has how much percentage of PO2 = 30 % 261.Craniophrangioma ablation is done in a woman due to tumor, what is required for ovulation = HMG followed by HCG 262.Which drug does not cause teratogenicity = flouxeten 263.In chronic intra vascular hemolysis = Dec heptoglobin 264.Severe bradycardia and low pulse, oculocardiac reflex is stimulated, what is the immediate treatment = Stop stimulus 265.Syncope is not a recognized feature of = Haemophilia 266.The most common cause of anemia in pregnancy = Haemodilution 267.Suprapubic incisioňis given by a surgeon, artery injured in the suprapubic fat is = inferior epigastric 268.Pudndal nerve does NOT supply = Posterior vaginal fornices

Session 6

86

( its one of the most frequently repeated question, so memorize it by MNEMONIC; P repels P ie PUDENDAL does not supply Posterior fornix ) 269.Least likely cause in 17 to 20 weeek abortion = Fetal cytogenetic abnormality 270.Regarding hylaine cartilage = Collagen fibers are not visible 71.How can rubella infection be prevented = Anti rubella vaccine ( for prevention child bearing females should be vaccinated) 272.Insoluble storage form of iron in iron overload = Hemosiderin 273.Torch can causes congenital abnormalities except = Pneumonia 274.The gland having more serous portion is = Parotid gland 275.True about kidney and relations of different structures passing through it is = Left renal vein is anterior to aorta and left renal artery 278.Nerve supply of urogenital triangle = Ilioinguinal nerve 279.Nerve supply of detrusor muscle = S2, S3, S4 280.Women having lack of vagitable diet resulting deficiencies = Anencephaly 281.Visual impulse is associated with = Dissociation of rhodopsin to retinal and opsin 282.most dangerous type of hepatitis for pregnt ladyin near future = Hep E ( so the rule is , lethal in pregnancy is Hep E while most lethal if pregnancy is not mentions, is Hep D , ie D for Deadly) 283.premalignant leision of vulva = Condyloma 284.breast lymphatic drai upper outer qud = ant axillarynodes ( also remember another frequently tested fact that, sentinel lymph node biopsy is done in order to decrease needles axillary clearance in lymph node negative patients Ref; Oxford medicine) 285.prolactin regultd by = Dopamine ( Dopamine is also called prolactin inhibitory hormone as it inhibits Prolactin) 286.regarding levator ani muscle = Supplied by pudendal nerve 87.UV prolapsed most common complication = Chronic cervicitis and metaplasia 288.regardingg menopause correct one is = Inc gonadotophins ( Dec Estrogen and increase FSH and LH ) 289.regarding sorbitol = Alcohol sugar 290.True about uremic nephropathy = Normocytic normochromic anemia 291.A disc is herniated, it will affect = spinal nerve+ posterior root 292.Phrenic nerve is related to which artery = Pericardiophrenic artery 293.Which immunoglobulin is present in RH incompatibility = IgG

Session 6

87

294.The most common route of transmission of HBV and HCV is through = Through sharps and needles 295.Thirst is increased by = decrease in plasma volume 296.Which mechanism is involved in carrying the particle inside the cell when actin, myosin and clathrin are involved = receptor medicated Pinocytosis (whenever u see clathrin, in the stem , click receptor mediated endocytosis/pinocytosis, and its most common mechanism for cholesterol uptake by cell) 297. Estrogen’s side effect = Thromboembolism ( while low dose estrogen containing pills cause hepatic adenoma, also note that hepatic adenoma is caused by a class 3 antiarrythmic ie AMIODORONE) 298.Left renal vein = Anterior to aorta and left renal artery 299. local metabolites in skeletal muscles = Decrease TPR ( note that Autoregulation is mainly by Local metabolites in skeletal muscles brain and coronary arteries) 300.Below knee amputation irregular mass after sometime = Neuroma

Session 7

88

Accept the challenges so that you can feel the exhilaration of victory. George S. Patton 1.Eating undercooked and raw meet can cause infestation by = tape worm (Note that, taenia solium via uncooked pork , T.Saginata via uncooked beef & fish tape worm (Diphyllobothrium latum) via uncooked fish) 2.A postmenupasual women suffered fracture due to osteoporosis her Ca is decreased and phosphorus is increased , she has = inc PTH 3- Mass movement is affected in colon due to problem in = Auerbach plexus (while submucosal plexus are for secretions ) 4- In an adult Ileactomy and jejunectomy has been done. He has diarhoea and 15-20 bowel sounds per minute. The most likely cause of this decreased transit time in the intestine is due to = secreroy plus osmotic 5- treatment of vWD = Cryoprecipitate ( cyopreciptate should be pronounced as cryoprecipit-8 as it contains factor 8, and in vwd factor 8 is depleted ) 6- The head of femur in adult is supplied by = Medial and lateral circumflex artery ( while in children its supplied by OBTURATER artery) 7- Head of humerus is supplied by = Anterior circumflex ( while fracture will damage posterior circumflex artery) 8. A man having recurrent bacterial infections with streptococci also having diarrhea which on examination showed giardiasis, further exam revealed normal T cells but ↓B cells levels of igM,igG and igA were given = Bruton's agammaglobulinemia ( this scenario is repeatedly encountered so plz NOTE THE WORD GIARDIASIS , and that will help u in memorizing stem.) 9- A 6 years child who develops muscular weakness. Anti voltage gated calcium chanel antibodies are presen = Lambert Eaton syndrome 10- Infarction of posterior 1/3 of interventricular septum.artery involed -.RCA ( while the anterior 2/3 of septum is supplied by left ant descending arter ie LAD) 11- Right border of heart is formed by = right atrium ( IF ASKED in xray the right border is formed by rt atrium plus SVC, AND if this option is not present then click only SVC ) 12. which is a small blue cell tumour asociated with large amount of catecholamines in child = Neuroblastoma

Session 7

89

13- which of the following prevents muscle tear under high pressure = Golgi tendon organ (note, when the muscle is in tension = Golgi tendon starts firing and muscle get relaxed in response ) 14-Which of the following ulcer is caused by synergistic infection by microaerophilic non hemolytic streptococci and aerobic hemolytic staphlococci = Meleney's ulcer 15- Cholecystectomy is done, where to put the drain = Subhepatic space 16-Esophagectomy done stomach is mobilized to upper portion of thorax ..To anastomosis with phayrnx, Which artery is responsible for stomach supply now = right gastroepiploic artery ( if this artery is not mentioned in options then click RIGHT GASTRIC artery) 17-common Postop infection in hospital cause by which organism = Staph Aureus ( second most common is E coli) 18-clawhand produced by unopposed action of = extensor digitorum and flexor digitorum profundus 19-Recurrent renal stone associated with which amino acid abnormality = Lysine 20- Rotator cuff is formed by = supraspinatus, infraspinatus, teres minor and Subscapularis ( memorize by Mnemonic SITS ) 21-If FACTOR IX not available = FFP (Note that if asked which is used as immediate antidote to WARFARIN then as is FFP not Vit K, because vitK Needs time to work and not imp for immediate action ) 22-which of the following drain into superior mesenteric lymph node = Jejunum ( also remember that jejunum is supplied by both GREATER , LESSER Splanchnic nerve while stomach by Greater splanchnic nerve ) 23-Drug which is strong analgesic but not a good anesthetic -nitrous oxide 24- peptic ulcer profusely bleeding from posterior wall of duodenum...which artery involved = gastroduodenal artery ( memorize this artery by the site of ulcer i-e ulcer is common in Gastric n Duodenal parts of gut so artery should be GASTRO-DUODENAL in case of bleed) 25- Lumbar puncture = L4 L5 (While L3 L4 should be clicked if L4 L5 is not present in option ) 26-Seminoma is a tumour in male, its equivalent when occurs in ovary then its called = dysgerminoma 27- Which of the following is mostly premalignant in the oral cavity Erythroplakia

Session 7

90

28- A female pt was having generalized pain, and has been taking NSAIDs for a long time. Now developed swelling n big toe, wht will b present in fluid aspirate = mono sodium urate crystals 29- About small intestine true is small intestine = 6-8meters long 30. Compression effects on esophagus by which organ enlargement = Left Atrium ( as it’s the most posterior part of heart and lies almost over esophagus ) 31. Prostatic carcinoma spread to skull bones through = Vertebral plexus 32. A Person wid malignant melanoma. Risk factor = Xeroderma pigmentosum 33.Apoptosis = cell shrinking ( if councelman body is mentioned in options then click that ) 34.Open wound healing includes = myofibroblast 35.An obese man with h/o atherosclerosis underwent laparotomy due to bleeding in abdomen n during surgery, his small gut was purple n sup mesenteric artry was blocked , so what is dx =wet gangrene 36. Aortic aneurysm which structure not compressed = Phrenic nerve 37.regarding cimitidine intake what effect causes = inibition of hepatic enzymes 38.ewing sarcoma occurs which site of bone = diaphysis 39.K in diet is excreted in which part of nephron = distal tubules (Note that, 67% of the filtered K is reabsorbed along proximal tubule , but Distal tubules and collecting ducts either reabsorb or secrete K , depending on dietary K intake, the excretion ranges from 1% to 110% . if both DCT and collecting duct are present in options prefer, distal convoluted tubules) 40.after non-lethal MI in 4 minutes = arrhythmias 41.Adductor muscle hamstring muscle supplied by which nerve = tibial 42.a statistical test observed differences b/w 2 means been by chance = variance 43..max pressure in aorta during which phase = max during slow ejection phase ( while lowest in isovolumetric contraction) 44.remant of geburnaculum = Ovarian ligament and round ligament of uterus 45.left renal vein relation to aorta = LRV is anterior 46.cause of thromboembolism = prolonged immobilization(most common) 47. commonest cause of shift of K from intracellular to extracellular = strenuous exercise (releases K from skeletal muscle) 48. damage to medulla oblongata during hanging = odontoid 49. sternocleidomastoid causes = movement of head in opposite direction against force(resistance) 50-platelets have life span of = 10 days (note that platlets Count increases after spleenectomy the first effect we see after

Session 7

91

spleenectomy is Increased Platelet count ie thrombocytosis as the sequestered RBCs are put into circulation, and the HOWEL JOLLY BODIES are after that) 51.multiple bone fractures history,blue sclera which products enzyme is defiecient =Collagen (the scenario is osteogenesis imperfecta) 52..MELAS syndrome histological finding = Red ragged fibers 53..Giant Cell Tumors histo finding = Soap Bubble appearance 54 .Platelets cannot be given in = Spleenomegaly (if asked contraindicated then Platelets are Contraindicated in Itp) 55.Bipolar cells are found in = olfactory 56.lobes of breast through many terminal ducts open into = lactiferous sinus 57.Left adrenal gland drains into = Left renal (same is the case for left testicular vein that also drains into left renal vein ) 58.Free radical injury produces pathology by affecting = DNA 59.A 60 year old woman known case of Ischemic heart disease, Carcinoma breast has now come with osteoporosis. Which drug you will prescribe to treat her = Raloxifene 60.In osteoporosis which of the fallowing is defective = Ostoclasts 61.Fast impulse passes through = Pukinje fibers 62.Internal laryngeal nerve passes through = thyroid and hyoid 63.liver development = from ventral mesentery proximal Foregut 64.patient developed hemolysis due toG6PdD deificiency nxt happen = self limiting 65.required for transport of B12 in blood = TC2 66. Anemia with Blast Cells in peripheral blood = Erythroblastosis fetalis ( note the point ie BLAST in the question and BLAST in the answer ) 67.Deep to post digastric and near palatoglossus a structure runs obliquely upwards = Lingual artery 68.hip extension and leg flexion by = semitendonosis 69.Universal donor = O negative ( while universal recipient is AB positive ) 70.Which structure is formed by the end of first week = blastocyst 71.-injury above sacral spinal cord result in = Spastic bladder (note that UMN lesion ie the lesion above sacral spine is = spastic bladder while due to lesion of the sacral spinal cord segments or sacral spinal nerve roots , means LMN lesion result in so called Atonic bladder. Neurogenic bladder is the term used for both spastic and atonic, so if clear-cut point is not asked then , prefer neurogenic bladder) 72.Cell membrane structure primarily maintained by = Cholesterol bilayer

Session 7

92

73.Enzyme may fall in liver injury = Pseudocholinesterases [decrease in liver disease(hepatitis and cirrhosis)] 74. which part of liver with poor supply = poor oxygenation of zone 3 ( note that zone 1 is richly supplied with blood and is mainly effected by VIRUSES & INGESTED TOXINS, while zone 3 is poorly perfused and mainly affected by metabolic toxins ) 75.trachea is = has 16-20 incomplete rings 76.hyper eosinophilia occurs in = hodgkin ( due to IL5, that stimulates the esinophills production ) 77.corticopsinal track ends at which level = spinal cord (if asked anterior corticospinal tract or anterior spinothalamic tract ends at ?? the ans for both is = midthoracic region ) 78.warthin starry stain used for = spirochete( S for starry and S for spirochete) 79.phrenic nerve runs on left side with = pericardicophrenic artery 80. hypovolaemia with hypotension isoflurane shud not be given,why? b/c it is = Potent vasodilator 81. Otic ganglion location foramen was asked = inferior border of foramen ovale 82.Epitheluim of preterminal bronchiole/ conducting zone = Pseudostratified columnar cilliated...with goblets in larger bronchiole (in resp bronchioles> cuboidal, then simple squamous upto the alveoli] {clara cells in terminal bronchioles} ) 83.muscle which causes flexion of elbow & supplied by radial nerve= Brachioradialis 84.hormones increased during sleep = Growth hormone (note growth hormone increases during sleep but if asked During sleep inc level than awake click cortisol) 85..chest pain and fever not related to respiration = Myocarduim 86..in later stage of pregnancy oestrogen and progesterone produce by = placenta 87..A patient has hypercouagable state having def of = factor v(facror V Leiden) 88..difference BTW skeletal and smooth muscle = ca calmodulin 89..Valve involved in Limb sack endocarditis = Mitral & aortic 90..Abdominal angina cause by obstruction to = Sup mesenteric 91..middle colic is the branch of = Superior mesenteric artery 92..geniculate ganglion is located in = facial canal 93..taste sensation is carried to the cortex through = thalamus

Session 7

93

94..two point discrimination = merkel disc (If asked ,Maximum 2 point discrimination at then= Lips( TIP OF TONGUE BEST IF THERE ) while if the stem is “Maximum distance of 2 point discrimination at” then = BACK OR scapula) 95..oesophageal atresia = drooping of saliva (indicating factor) 96..Aluminium hydroxide in = delays gastric emptying (causes constipation) 97..long refractory period = heart ( and due to this long refractory period cardiac muscles cannot be tetanized ) 98..defeciency of glucose 6 Phosphatase causes = hypoglycemia.. 99. Increase GFR causes inc absorption of salt and water from = PCT 98.Infection in cavournes sinus = INF OPTH VEIN (inferior ophthalmic vein for infection and superior for thrombosis ) 99..cause of damage to lateral rectus = 6th CN 100..which one is the DNA virus = Infectious mononucleosis 101..Arterial emboli mostly arises from = Mural left atrium 102.Most of girls have amenorrhea during lactation, cause is = It inhibits gonadotropin releasing hormone ( ie FSH and LH) 103.True about breast = Arise from mammary buds present on the memory line 104.A patient having AIDS may also have one of the following with AIDS = cervical Carcinoma 105.Severe hypomagnesemia may result in one of the following = Hypocalcemia (note that hypomagnesaemia causes hypercalcemia while severe hypomagnesaemia causes HYPOCALCEMIA ) 106.The time interval between 1st and 2nd heart sound is almost equal to = Ventricular systole 107.The side effect of chewing betal nut = submucosal Fibrosis 108.Aldosterone is secreted in response to = inc in plasma potassium (● inc in plasma potassium is the most potent stimulant for Aldosterone, ● while the most potent stimulant for ADH is nausea & ● most potent stimulus for renin is sympathetic drive ) 109.More mitochondria are present at the apex of which of the following = ciliated cells 110.Physiological jaundice = After the 2nd day of birth of the baby (JAUNDICE on day 1 is always pathologic while in preterm babies, even jaundice on day 2 is pathologic ) 111.Parents are informed about the disability of their child due to a debilitating disease. The first reaction of the parents is = Denial

Session 7

94

112.shortening of chromosome in mutation....enzyme responsible is = telomerase ( this enzyme is responsible for the specific number of divisions a cell should go through, and with every division cell shortens and apoptosis is the end result, if this mechanism is defective , cell will evade apoptosis and proliferates to become neoplastic) 113.Regarding red infarcts = dual blood supply 114.pressure is dissipated at = arterioles capillaries 115.highest cross sectional area is of = capillaries 116.After 6 day of MI shock occurs d/t = pericardial tamponade 117.Highest magnesium stores r present in = bone 118.diabetic pt loss of sensation n vibration n has ulcer in foot mechanism behind = angiopathy and neuropathy 119.turbulence of blood in anemia occur d/t = dec viscosity 120.regargding breast atrophy oin young is caused by = Estrogen n progesterone ( so many explanations for the breast atrophy, but I am always in the favor of estrogen and progesterone both as one causes proliferation of glands and other causes proliferation of stroma, and for atrophy of breast, both glands and stroma should be involved) 121. Fracture of surgical neck of Humerus damage to = Axillary Nerve ( note that deltoid is supplied by axillary and damage to it result in loss of abduction, ●supraspinatus - by suprascapular nerve = 15 to 30 degree abduction ●Deltoid – by axillary nerve = upto 90 degree abduction ● trapezius – by accessory nerve = above 90 degree abduction) 122.. Period where heart is susceptible to fibrillation = At the end of action potential 123. Aortic pressure is lowest during = isovolumic contraction ( about 70mmHg) 124. Labrynthine artery is the branchof = basilar (it arises form AICA but if not in options click Basilir ) 125. Lady after cholecystectomy , having microcytic anemia = Blood loss

CPSP Demo

95

try not to mess with the key ) 1.Patient has aphasia and facial nerve palsy on same side the artery involved is = middle cerebral artery 2.Regarding l and d sugars = D sugars has oh on right side 3.A patient has non-reactive HbSAg , non-reactive HbeAg , and reactive antiHBC = transient resolving stage 4.Urine examination of a patient with DM demonstrates ketones bodies,, what’s the mechanism of formation of these ketone bodies = insulin deficiency 5.Which is not a tumor marker = acid phosphatase 6.Most sensitive cells to hypoxia = neuron 7.Tyrosine derivative doesn’t include = prolactin 8.Organ having least chances of infarction = liver 9.Primary malignant melanoma of choir dost commonly metastasize to = liver 10.Which is the site of fusion of binocular vision = visual cortex 11.A newborn baby with hydrocephalus ha swelling in lumbosacral lesion of spine which contain neural tissue in it what’s diagnosis = meningomyelocel 12.Regarding trigeminal ganglion the correct statement is = it’s like a posterior root ganglion of spinal cord 13.The total power of the eye is 59 diopters the main role in maintaining this is played by = anterior surface of cornea 14.In case of six days typhoid the test of choice is = blood test 15.After giving blood transfusion the patient develops hypersensitivity reaction which of reaction is this = type 2 hypersensitivity 16.For diabetic patient the best antihypertensive is = captopril 17.Patient with bronchial asthma the best antihypertensive = verapamil 18.Mesothelioma is associated with = asbestos 19.Nerve injured in anterior dislocation of shoulder is = axillary nerve 20.Hypoglycemia increases = secretin 21.Drug that increases extracellular K moves K out of the cell = angiotensin 22.Dark urine after starting ATT = rifampin 23.Thirst is decreased by = baroreceptor efferent 24.A diabetic patient is advised fasting blood glucose level at what value doctor is confusing and needs further evaluation with Glucose tolerance test = 7 mmol/l

CPSP Demo

96

25.A female has goiter and she feels difficulty in breathing while lying down, what type of goiter she has = retrosternal 26.A patient with enlarged parotid gland with pain in this region, which nerve is carrying pain fibers from parotid gland = auriculotemporal nerve 27.A patient develops lesion in the caudate nucleus , what is the most common clinical feature indicating this lesion = chorea 28.Regarding the vibration sense what is INCORRECT = its receptor is messner corpuscle 29.Defect in the formation of bulbus cordis results in all of the following except = ASD 30.A 40 weeks pregnancy lady has irregular uterine contractions, which drug will be beneficial for this lady = oxytocin 31.A patient with aplastic anemia is given anti lymphocytic globulin, one week later she developed skin rashes. Mechanism for this presentation is = type 3 hypersensitivity 32.Which of the following is the most common cause of osteoporosis except old age = Cushing disease 33.A male patient with the antibody again FSH receptors which one will be lower than normal I this person = sperm count 34.The most common site of fertilization ins human is = fallopian tube 35.Structure entering the thoracic inlet is = aorta 36.Needle inserted at the 5th intercostal space on the left side of sternum pierces = intercostal muscle 37.Somatostatin decreases the secretion of which hormone = insulin 38.Gastric acid is stimulated by = Gastrin 39.Regarding SA node = automatically generates impulses, 40.During stretch in the skeletal muscle, which of the following change will occur in nuclear bag fibers = they increase impulse generation 41.Pulmonary artery pressure increases in= hypoxia 42.Which of the following structure in the mouth is derived from ectoderm = epithelium of the parotid gland 43.Oxygen level in the blood will decreases in = Hypoxic hypoxia 44.A 6 cm lump in the breast is removed, four lymph nodes and skin were involved, biopsy decided invasive ductal carcinoma , this tumor is called high grade because of = Pleomorphism 45.Diagnostic finding on investigation of metabolic alkalosis is = HCO3 more than 24meq/liter

CPSP Demo

97

46.In a patient with sympathetic stimulation what will be the effect on his heart = dec AV Nodal delay 47.What is the nerve supply of the tip of nose = ophthalmic 48.A patient old male has central retinal artery occlusion what could be the most likely cause = embolism 49.A patient with recurrent epistaxis, lab investigations show decrease factor 2, 7 , 9 , 10 , protein c and s , what will be the most likely cause = vitamin k deficiency 50.Most common malignancy in person suffering from nuclear blast = hematopoietic 51.In turner syndrome the genotype is = XO 52.Vertebreae are derived from= slcerotome 53.Regarding filliform papillae = they are present on the dorsum of the tongue till the tip 54.A patient is unable to extend metacarpophalangeal joint , abduct and extend his thumb, but intact sensation, which nerve is damaged = radial nerve 55.A young patient blood pressure is 150/95 his serum renin level is higher than normal which one is the stimulus for higher renin = increase sympathetic stimulation via renal nerves 56.Which one is the characteristic of IgM = it’s a largest molecule 57.A patient with chronic renal failure needs transplantation. For this reason HLA matching is required , which is the best site for sample for HLA matching = WBC 58.A young child has increase BP in upper limb while decrease BP in lower limb, the pulse is also weak, what is the most probable diagnosis = Post ductal coarctation of aorta 59.Penetrating injury in the 4th left intercostal space just lateral to the sternum will damage, which one of the following structure first = intercostal membrane 60.Best pulmonary function test for asthma is = FEV1 61.Nerve injured in anterior dislocation of shoulder joint is = axillary 62.Presene of pancreatic tissue in the gastric mucosa is = choriostoma 63.Least common site for ectopic pregnancy is = ovary 64.Least chances of renal stone is associated with = hyperlipidemia 65.Which enzyme is released form small intestine = enterokinase 66.A patient has finger like projection on upper eyelid, histopathlogical reports show epithelial cell along the fibrous element what is the most probable diagnosis = papilloma 67.Parotid gland is supplied by = GVE

CPSP Demo

98

68.Which one is most strong antioxidant = Glutathion 69.A patient develops paralysis of the left side of the body , after few days his behavior for pain is changed, and eh become angry after touching, where could be the lesion = right thalamus 70.Most early diagnosis of vita min a deficiency = night blindness 71.Patient with bone pain, normal calcium and in alkaline phosphatase = Paget disease 72.which of the following structures is produced in 3rd week of development = Heart tube 73. Antibodies are produced by = Plasma cells 74. Patients come with deviation of tongue to right side. Decreased sense of touch and vibrations, the artery commonly involved in brain is = Anterior Spinal 75.Patient with injury to left 8th cervical segment of spinal cord will not show following sign = Dec sense of pain and temperature below the lesion on same side 76.Foetal period starts after which week = 8th 77.Patient with old history of adenocarcinoma of colon operated for polypectomy, on histologic evaluation pathologist labeled it as benign growth with no chances into malignant transformation, it would be = Tubular adenoma 78.Soldier comes with heavy bleeding. The ideal fluid replacement would be = Whole blood for 3 days 79. A Patient has anemia, hyper-segmented Neutrophils on peripheral blood examination & neurological manifestations. Which type of anemia he is suffering from = Pernicious anemia 80.Heme binds with = Haptoglobin 81.A pregnant lady develops pain few hours after delivery. E.C.G. shows S1, Q3 and T3. What is another investigation most appropriate for the diagnosis of this case = Gallium Scan 82.In a patient diarrhea are corrected after the fasting. Which is most likely type of diarrhea = Osmotic 83.Councilman bodies are formed in the process of = Apoptosis 84.A middle aged male presents with joint pain since one week. His serum Uric Acid level is 156 mg/dl. What is the best investigation for confirmation of the disease = Synovial fluid for polarized light 85.A patient comes to you in ill condition. His blood pH is 7.3 PCO2 is 44 mmHg & HCO3 is 17 meq/L. What diagnosis you are thinking of this patient = Metabolic acidosis

CPSP Demo

86.During general anesthesia Halothane is given in combination with which of the following drugs = Nitric oxide 87.Most common cause of the pelvic inflammatory disease is = Chlamydia 88.Which cranial nerve carry parasympathetic fibers = 3, 7,9 ,10 89.Which is the characteristic feature of rheumatic fever = mitral stenosis 90.Axillary and pectoral group of lymph nodes drains = upper and lateral quadrant 91.A 20 weeks pregnant lady presented with BP of 140/95 she complains of vomiting and headache, what is the most probable diagnosis = pregnancy induced hypertension 92.Brucellosis is transmitted by = unpasteurized milk 93.A patient develops sudden breathlessness , hypotension , severe itching and other features of anaphylactic shock ,which drug will be beneficial for this patient failing heart = adrenaline 94.Parasympathetic stimulation causes = opening of intestinal sphincters 95.A patient with trauma of the head developed cerebral edema, which is the cause of inc respiratory rate in this patient = raised ICP 96.Secondary hyperaldosteronism can occur due to = inc renin 97.All of the following mechanisms are involved in development of diabetic gangrene EXCEPT = Arterioscierosis leading to reduced blood flow to foot 98. Blood brain barrier is formed by = Capillary endothelium 99. On H & E staining a student sees the hallow structure around the nucleus. What it could be = Lysosomes 100.A middle aged male has non-healing wound for long time in presence of regular dressing; random blood sugar is 130 mg/dl. What could be the cause of delayed healing in this patient = Vascular insufficiency 01.Which of the following is diagnostic of granuloma = Epithliod cells 02.Medial orbitotomey is done in a patient with tumor in orbit. Now he is complaining of numbness at upper part of head up to vertex & medial part but medial part is intact. Which of the following nerves is damaged = Supraorbital nerve 03.In inferior wall Myocardial Infraction, which artery should be blocked = Right marginal artery 03.A young women, complains of Dry mouth & Dry eyes. Which investigation will give clue to her diagnosis = Anti - SS A/B 04.Esophagus histology = Has str. sq in upper 1/3 05.Example of carrier mediated counter transport = Na - glucose transport

99

CPSP Demo

100

06.Following is true regarding sterilization = Autoclaving is heating objects at 121 degree C at 15 psi for 3min 07.Pt complains of chest pain for more than 30 min. ECG shows changes in V1V4. It denotes = Anterior wall MI 08.Depolarization of cell is maintained by = Na influx 09.Spinal cord ends at the level of lower border of = L2 vertebra 10.Dry mouth increases all except = Plasma volume 11.Urine Examination of a patient with diabetes demonstrates. Ketone-bodies. What is the mechanism of formation of these ketone-bodies = Insulin deficiency 12.A lady received DES during pregnancy for prevention of the abortion. Her baby will be on the risk of = Clear cell carcinoma of Vagina 13.Prenatal chromosomal abnormality can be detected at = 14-18 weeks 14.All of the following mechanisms are involved in development of diabetic gangrene EXCEPT = Arterioscierosis leading to reduced blood flow to foot 15.A patient presents with dislocation of hip after an accident. Which of the following muscle group is involved =Gluteus Maximus + Minimus + Medius 16. No question,  = just chill 17.A patient has history of recurrent abortion. Which of the following enzyme analysis will give help to diagnosis = Progesterone 18.A patient presents with secondary amenorrhea. One year back she has history of P.P.H for that she received six pints of blood what is most probable cause of her menstrual problem = Sheehan's syndrome 19.A 60 years old women is brought with bleeding per vagina. She is diabetic & hypertensive & one year back she was diagnosed as having carcinoma of breast. What can be most probable diagnosis for this bleeding p/v = Carcinoma of endometrium 20.A Patient is suffering from carcinoma of rectum. He develops pain in the posterior aspect of thing due to involvement of nerve. Which nerve supplies the posterior aspect of thigh & that can be involved in the malignancy of rectum = Sacral nerves 21.A semiconscious patient is brought to emergency department. He has history of taking some unknown drug. NaHCO3 reverses the action of drug. Which drug he has most likely taken = Phenobarbital 22.A lady with 26 weeks gestation has uterine height of 24 weeks. On sonography there is no fetus & snowstorm appearance. Which of the following probably associate with it = Karyotype-46XX

CPSP Demo

101

23.Normal female pelvis has = Oval inlet 24.Ascent of the horse shoe shape kidney is prevented by = inf mesenteric artery 25.Which of the following is characteristic of polycystic ovary disease = Inc L.H 26.Milk is notoriously deficient in = pantothenic acid 27.Down syndrome Is characterized by= mental retardation and abdominal hernia 28.A lady undergoing surgery for cranyophargioma , what you could expect to be present in this lady = Hyperprolactemia 29Regarding nitroglycerine all are true except = increase heart rate 30.A person presented with the loss of visual acuity which artery is most likely affected = posterior cerebral artery

Good Luck for exam Regards; Tan Veer Ahmad

View more...

Comments

Copyright ©2017 KUPDF Inc.
SUPPORT KUPDF